Download as pdf or txt
Download as pdf or txt
You are on page 1of 155

Introduction

NOTE: The intent of MedicalPearls Publishing Board and Certification Tests is to


provide practice questions and explanations to assist you in identifying and
resolving knowledge deficits. These questions and explanations are not intended to
be a source of the knowledge base of medicine. The authors or editors do not
warrant the information is complete or accurate. The reader is encouraged to verify
each answer and explanation in several references.
All drug indications and dosages should be verified before administration.
Copyright 2016 MedicalPearls Publishing, LLC.
All rights reserved, including the right of reproduction, in whole or in part, in any
form.
These questions and answers will help improve your fund of knowledge. Our
questions and explanations are designed to help you identify and resolve knowledge
deficits. To achieve this goal the text is written in a multiple-choice format with
answers and explanations. Explanations contain additional information intended to
reinforce your knowledge. Emphasis has been placed on covering facts that are
easily overlooked, quickly forgotten, and often show up on exams.
WE APPRECIATE YOUR COMMENTS!
Please send suggestions, corrections, and ideas for improvement to:
editor@medicalpearlspub.com

Table of Contents
Introduction
Chapter 1 ( Questions 1 - 15 )
Chapter 2 ( Questions 16 - 30 )
Chapter 3 ( Questions 31 - 45 )
Chapter 4 ( Questions 46 - 60 )
Chapter 5 ( Questions 61 - 75 )
Chapter 6 ( Questions 76 - 90 )
Chapter 7 ( Questions 91 - 105 )
Chapter 8 ( Questions 106 - 120 )
Chapter 9 ( Questions 121 - 135 )
Chapter 10 ( Questions 136 - 150 )

Question 1: MRI signal with early traumatic cord injury has what
appearance?

Choices:
1. Both T1 and T2 signal is increased
2. T1 signal is isointense to cord; T2 signal is increased
3. T1 signal is hypointense; T2 signal is hyperintense
4. The earliest finding is cord swelling but no signal change

Answer: [2] - T1 signal is isointense to cord; T2 signal is increased


Explanations:
The signal changes are due to cord edema, not hematomyelia, which is not
commonly present
As the edema resolves, the cord appearance improves
Cord swelling may also be present

Study Topic: Spinal Cord, Injuries


We update our content daily based on user feedback. Please click here to submit
your suggestions or if interested in becoming an author or editor, please contact us
at editor@medicalpearlspub.com.

Question 2: 18-F FDG is


Choices:
1. Distributed to the brain as a function of blood flow only
2. Is a K+ analog
3. Has a half-life of 59 minutes
4. Is taken up by myocardium normally

Answer: [4] - Is taken up by myocardium normally


Explanations:
Brain distribution is a function of both cerebral blood flow and metabolic
activity; this is why gray matter takes up more tracer than white matter
FDG is a glucose, not a potassium, analog
FDG half-life is 109, not 59, minutes
Normal myocardium may, though doesn't always, take up FDG. It is in part
related to the blood glucose level

Study Topic: Fludeoxyglucose (18F)


We update our content daily based on user feedback. Please click here to submit
your suggestions or if interested in becoming an author or editor, please contact us
at editor@medicalpearlspub.com.

Question 3: Direct visualization of the peritoneal cavity is accomplished by:


Choices:
1. Colposcopy
2. Hysteroscopy
3. Laparoscopy
4. Dilation and curettage

Answer: [3] - Laparoscopy


Explanations:
A laparoscope is used to directly visualize the peritoneal cavity

Study Topic: Laparoscopy


We update our content daily based on user feedback. Please click here to submit
your suggestions or if interested in becoming an author or editor, please contact us
at editor@medicalpearlspub.com.

Question 4: Which of the following is NOT an advantage of MRI over CT?


Choices:
1. Multiplanar imaging without repositioning or post-processing
2. Delineation of soft tissue lesions
3. Imaging of vascular lesions/ pathology
4. Imaging of solidly calcified structures

Answer: [4] - Imaging of solidly calcified structures


Explanations:
Imaging of solidly calcified structures (eg, bone, cartilage) is better with CT
scanning.
Other advantages of CT over MRI include lower expense, faster imaging time,
and the ability to image claustrophobic patients and patients with various
electronic devices (e.g., pacemakers).
Other advantages of MRI over CT include less need to administer IV contrast,
use of an IV contrast agent less damaging to the kidneys.
Previously, CT had the advantage over MRI in evaluating structures that moved
(e.g., abdominal imaging with respiratory and peristaltic movements) but
improved sequences and technical improvements have enabled both CT and MRI
to image even the beating heart--outstand spatial and temporal resolution.

Study Topic: Magnetic Resonance Imaging (MRI)


We update our content daily based on user feedback. Please click here to submit
your suggestions or if interested in becoming an author or editor, please contact us
at editor@medicalpearlspub.com.

Question 5: Which of the following is not a feature of hemangioma of the


respiratory tract?

Choices:
1. It is more common in the trachea and mainstem bronchi
2. There may be multiple and bilateral tumors
3. They may be associated with Osler Weber Rendu syndrome
4. The treatment of choice is embolization

Answer: [4] - The treatment of choice is embolization


Explanations:
Hemangiomas in the tracheo-bronchial tree are usually solitary and can produce
airway obstruction.
Diagnosis is by bronchoscopy and is best treated with surgery, laser, and/or
radiation.
They may be associated with the Osler Weber Rendu syndrome.

Study Topic: Hemangioma


We update our content daily based on user feedback. Please click here to submit
your suggestions or if interested in becoming an author or editor, please contact us
at editor@medicalpearlspub.com.

Question 6: Fistulae are MOST common in which gastrointestinal (GI)


disease?

Choices:
1. Crohn disease
2. Ulcerative colitis
3. Ischemic colitis
4. Antibiotic-associated colitis

Answer: [1] - Crohn disease


Explanations:
The occurrence of a fistula or abscess of the perineum or vagina should always
raise the suspicion of irritable bowel disease, particularly Crohn disease, even in
the absence of bowel symptoms.
Rectovaginal fistulas have been reported in up to 23% of women with Crohn
disease.
Crohn fistulae can also include rectovesical fistula, enterocolic fistula,
enteroenteric fistula, and enterocutaneous fistula. Anorectal fistulae are also well
described.
Fistulae in Crohn disease are a result of the fact that the disease is transmural
and inflammatory.

Study Topic: Crohn Disease


We update our content daily based on user feedback. Please click here to submit
your suggestions or if interested in becoming an author or editor, please contact us
at editor@medicalpearlspub.com.

Question 7: Select the finding about MRIs in patients with migraines


demonstrated by the CAMERA study.

Choices:
1. Males with migraines with aura have increased prevalence of periventricular white
matter lesions (WMLs)
2. Females with common migraines have increased prevalence of deep WMLs
3. Cerebellar infarcts are not more common in patients with migraines
4. Periventricular WMLs are more common in patients with migraines and aura than
control patients

Answer: [2] - Females with common migraines have increased prevalence of deep
WMLs

Explanations:
The CAMERA study used MRI to assess deep and periventricular white matter
lesions (WMLs) in patients with and without migraines with and without aura
Cerebellar infarcts are more common in patients with migraines and even more
common in patients with aura once a month or more
Females with migraines had more deep WMLs than controls
Males did not show the same difference

Study Topic: Headache, Migraine


We update our content daily based on user feedback. Please click here to submit
your suggestions or if interested in becoming an author or editor, please contact us
at editor@medicalpearlspub.com.

Question 8: When removing a PICC from a patient, the tip breaks off and
the removed catheter is 1 cm shorter than the length inserted. What is the
appropriate action?

Choices:
1. Ambulate the patient
2. Perform vagal maneuvers
3. Place a tourniquet above the insertion site
4. Place the patient in Fowler's position

Answer: [3] - Place a tourniquet above the insertion site


Explanations:
A tourniquet should be applied above the insertion site.
The patient should be placed on bedrest, monitored, and kept calm. The patient
may be asymptomatic, or may have cyanosis, dyspnea, chest pain, hypotension,
tachycardia, increased CVP, loss of consciousness, arrhythmias, or a pulmonary
thrombus.
X-ray confirmation of the catheter fragment is performed, and surgical removal
may be necessary.
The same treatment is indicated for a peripheral catheter.

Study Topic: Percutaneous Central Catheter (PICC)


We update our content daily based on user feedback. Please click here to submit
your suggestions or if interested in becoming an author or editor, please contact us
at editor@medicalpearlspub.com.

Question 9: Which of the following arteries runs along the superior border
of the pancreas and enters the lienorenal ligament?

Choices:
1. Left gastric artery
2. Right renal artery
3. Left gastroepiploic artery
4. Splenic artery

Answer: [4] - Splenic artery


Explanations:
The splenic artery runs along the superior border of the pancreas and enters the
lienorenal (splenorenal) ligament.
It arises from the celiac artery and is superior to the pancreas.
Its branches include the greater pancreatic, short gastric, left gastroepiploic, and
posterior gastric arteries.
Unlike the splenic artery, the splenic vein is found posterior to the pancreatic
body. It is an ultrasound landmark for that structure.

Study Topic: Anatomy, Abdomen, Spleen, Arteries, Splenic


We update our content daily based on user feedback. Please click here to submit
your suggestions or if interested in becoming an author or editor, please contact us
at editor@medicalpearlspub.com.

Question 10: A middle-aged homeless man is presents with altered mental


status, seizures, and vomiting. Exam reveals multiple dental caries, and focal
neurological deficit, with no fever, head trauma, or neck stiffness. What should
the work up include next?

Choices:
1. Lumbar puncture
2. Non-contrast head CT
3. Contrast-enhanced head CT
4. Placement of ICP monitor

Answer: [3] - Contrast-enhanced head CT


Explanations:
Brain abscess is one consideration and presents sometimes without fever,
elevated WBC, and signs of meningeal irritation.
Brain abscesses develop by either contiguous spread or hematogenous spread.
Common sites of infection include sinuses, mouth/teeth, and ears. Contrastenhanced CT and MRI reveal a ring-enhancing lesion usually at the gray-white
interface with surrounding edema.
Treatment involves identification of organism through relief of mass effect with
aspiration vs open excision followed by IV antibiotics.
Brain abscess today is more common in the immunocompromised, including HIV
patients.

Study Topic: Abscess, Brain


We update our content daily based on user feedback. Please click here to submit
your suggestions or if interested in becoming an author or editor, please contact us
at editor@medicalpearlspub.com.

Question 11: An inadequate amount of amniotic fluid is termed:


Choices:
1. Polyhydramnios
2. Breech
3. Oligohydramnios
4. None of the above

Answer: [3] - Oligohydramnios


Explanations:
Oligohydramnios is an inadequate amount of amniotic fluid
It can result in poor lung development and fetal death
Oligohydramnios occurs in 4% of pregnancies

Study Topic: Oligohydramnios


We update our content daily based on user feedback. Please click here to submit
your suggestions or if interested in becoming an author or editor, please contact us
at editor@medicalpearlspub.com.

Question 12: A track between the surface of the skin and an underlying
structure is:

Choices:
1. Carbuncle
2. Hemorrhoid
3. Sinus
4. Abscess

Answer: [3] - Sinus


Explanations:
A sinus is a connection between the surface of the skin and an underlying
structure

Study Topic: Sinuses


We update our content daily based on user feedback. Please click here to submit
your suggestions or if interested in becoming an author or editor, please contact us
at editor@medicalpearlspub.com.

Question 13: What is a true statement about isolated right-sided


varicoceles?

Choices:
1. They compress easily
2. They are seen more commonly than varicoceles on the left
3. Their presence demands assessment of kidney and renal hilar regions for mass
4. They do not typically cause infertility

Answer: [3] - Their presence demands assessment of kidney and renal hilar
regions for mass

Explanations:
Isolated right varicoceles are uncommon and raise concern for a mass obstructing
the right gonadal vein or its drainage pathway to the IVC (renal vein). The more
common left-sided varicoceles drain directly into the IVC and thus don't
implicate renal mass lesions
If an obstructing mass is present, the varicocele will not be easily compressible
nor change significantly with Valsalva
A renal ultrasound to evaluate the kidney for a renal or hilar mass should be
performed.
Varicoceles are a correctable cause of infertility

Study Topic: Varicocele


We update our content daily based on user feedback. Please click here to submit
your suggestions or if interested in becoming an author or editor, please contact us
at editor@medicalpearlspub.com.

Question 14: Why is flowing blood hyperintense on time-of-flight


sequences?

Choices:
1. T2 effects
2. Coil choice
3. Saturation pulse
4. Flow related enhancement (FRE)

Answer: [4] - Flow related enhancement (FRE)


Explanations:
FRE signal arises from flowing blood. It is seen on GRE pulse sequences and
also in entry slices of multi-slice SE sequences.
FRE signal results from completely relaxed spins (called "unsaturated") that flow
into the plane or volume being imaged between RF excitations. Thus it is seen
more with faster flow and with thinner slices..

Study Topic: Magnetic Resonance Imaging (MRI)


We update our content daily based on user feedback. Please click here to submit
your suggestions or if interested in becoming an author or editor, please contact us
at editor@medicalpearlspub.com.

Question 15: Which are signs of brainstem herniation?


Choices:
1. Decerebrate or decorticate posturing
2. Fixed, dilated unilateral pupil
3. Severe obtundation, unable to arouse patient
4. All of the above

Answer: [4] - All of the above


Explanations:
All are signs of brainstem herniation

Study Topic: Herniation, Brainstem


We update our content daily based on user feedback. Please click here to submit
your suggestions or if interested in becoming an author or editor, please contact us
at editor@medicalpearlspub.com.

Question 16: Which is false about Gardner syndrome?


Choices:
1. It is a disorder that is genetically acquired
2. It is associated with osteogenic sarcoma
3. It is associated with multiple polyps
4. It is associated with desmoid tumors

Answer: [2] - It is associated with osteogenic sarcoma


Explanations:
Gardner syndrome is an autosomal dominant disorder that presents with multiple
polyps.
Extracolonic features of the disorder include osteomas of the skull, thyroid
cancer and a variety of cysts.
Gardner patients also develop desmoid tumors.
If the polyps are not removed, these individual develop colon cancer at a very
young age.

Study Topic: Gardner Syndrome


We update our content daily based on user feedback. Please click here to submit
your suggestions or if interested in becoming an author or editor, please contact us
at editor@medicalpearlspub.com.

Question 17: You can diagnose adenomyosis if the junctional zone


measures...

Choices:
1. 7 mm
2. 10 mm
3. 13 mm
4. Two of the above choices are correct

Answer: [3] - 13 mm
Explanations:
If the junctional zone is less than or equal to 7 mm, the finding is normal.
If the junctional zone is 12 mm or greater in thickness, you can diagnose
adenomyosis, independent of symptoms.
If the junctional zone is 8-11 mm in thickness, you can only say the findings are
consistent with adenomyosis "in the appropriate clinical context".

Study Topic: Uterus, Adenomyosis


We update our content daily based on user feedback. Please click here to submit
your suggestions or if interested in becoming an author or editor, please contact us
at editor@medicalpearlspub.com.

Question 18: Which of the following vessels drain directly into the inferior
vena cava?

Choices:
1. Renal veins
2. Left gastric vein
3. Inferior mesenteric vein
4. Superior mesenteric vein

Answer: [1] - Renal veins


Explanations:
Renal veins drain into the inferior vena cava.
Superior and inferior mesenteric veins and the left gastric vein drain into the
portal vein.
The portal vein drains into the liver and not into the inferior vena cava.

Study Topic: Anatomy, Abdomen, Kidneys, Veins, Renal


We update our content daily based on user feedback. Please click here to submit
your suggestions or if interested in becoming an author or editor, please contact us
at editor@medicalpearlspub.com.

Question 19: A patient has severe hypoxemia that is unresponsive to O2


therapy, a pulmonary artery wedge pressure less than 18 mm Hg, and CXR
findings of widespread alveolar consolidation with air bronchograms. What is
the most likely diagnosis?

Choices:
1. Pulmonary thromboembolism
2. Tuberculosis
3. Aspiration pneumonia
4. Acute respiratory distress syndrome

Answer: [4] - Acute respiratory distress syndrome


Explanations:
Atelectasis caused by ARDS leads to increased intrapulmonary shunting which
in turn causes an increased alveolar arterial gradient.
ARDS also causes diffusion abnormalities related to hyaline membranes.
CXR reveals bilateral interstitial infiltrates initially which progress to
widespread alveolar consolidation with air bronchograms.
The increase in permeability at the alveoli level leads to an influx of fluid, which
prevents oxygenation.

Study Topic: Acute Respiratory Distress Syndrome (ARDS)


We update our content daily based on user feedback. Please click here to submit
your suggestions or if interested in becoming an author or editor, please contact us
at editor@medicalpearlspub.com.

Question 20: About 13% of all surgically removed renal masses are benign.
These are most commonly...

Choices:
1. Angiomyolipomas (AML)
2. Oncocytomas
3. Juxtaglomular neoplasm (reninoma)
4. Options 1 and 2

Answer: [4] - Options 1 and 2


Explanations:
AMLs and oncocytomas are the most common benign tumors to lead to surgical
removal.
Although most AMLs are easily diagnosed as such because of their macroscopic
fat, about 5% have no visible fat on imaging. These tend to go to surgery.
Oncocytomas can mimic renal cell carcinoma on all imaging modalities.

Study Topic: Angiomyolipoma, Kidney


We update our content daily based on user feedback. Please click here to submit
your suggestions or if interested in becoming an author or editor, please contact us
at editor@medicalpearlspub.com.

Question 21: Pathogenesis of acute respiratory distress syndrome involves:


Choices:
1. Aspiration of nasopharyngeal flora while sleeping
2. Acute damage to alveolar capillary walls and epithelial cells
3. Saddle embolus formation
4. Atherosclerosis of main elastic pulmonary arteries

Answer: [2] - Acute damage to alveolar capillary walls and epithelial cells
Explanations:
ARDS is a noncardiogenic pulmonary edema resulting from acute alveolar
capillary damage.
Chemotactic cytokines that are released recruit neutrophils, which transmigrate
into the alveoli through the pulmonary capillaries.
Capillary damage results in leakage of protein rich exudate producing hyaline
membranes.
Neutrophil damage of type 1 and type 2 pneumocytes results in decreased
surfactant production, atelectasis and intrapulmonary shunting.

Study Topic: Acute Respiratory Distress Syndrome (ARDS)


We update our content daily based on user feedback. Please click here to submit
your suggestions or if interested in becoming an author or editor, please contact us
at editor@medicalpearlspub.com.

Question 22: In which of the following situations might the risk of


anaphylactoid reaction be greater than normal for IV contrast administration?

Choices:
1. Those with history of asthma
2. Those with history of allergies with respiratory symptoms
3. Those with no known medical history
4. 1 and 2

Answer: [4] - 1 and 2


Explanations:
Risk factors include allergy, asthma, renal insufficiency, various cardiac
pathologies (symptomatic CHF or angina, severe aortic stenosis, primary
pulmonary HTN, or severe cardiomyopathy) and anxiety.
This is true for both MR and CT intravenous contrast agents.

Study Topic: Magnetic Resonance Imaging (MRI)


We update our content daily based on user feedback. Please click here to submit
your suggestions or if interested in becoming an author or editor, please contact us
at editor@medicalpearlspub.com.

Question 23: A tennis player presents with elbow pain, on exam her pain is
exacerbated by forced wrist extension with the elbow extended; what is her
diagnosis?

Choices:
1. Lateral epicondylar tendinitis
2. Medial epicondylar tendinitis
3. Biceps tendinitis
4. Olecranon bursitis

Answer: [1] - Lateral epicondylar tendinitis


Explanations:
Lateral epicondylar tendinitis (tennis elbow) is an inflammation of the forearm
extensor muscles at their insertion site on the lateral epicondyle of the humerus,
Coven's test is positive when there is pain with resisted wrist extension with an
extended elbow
Golfer's elbow is inflammation of the flexor muscles at their insertion on the
medial epicondyle
Olecranon bursitis is a tender swelling at the posterior elbow caused by an
inflammation of the bursa over the olecranon process, often due to trauma or
associated with gout, infection or rheumatoid arthritis

Study Topic: Lateral Epicondylitis


We update our content daily based on user feedback. Please click here to submit
your suggestions or if interested in becoming an author or editor, please contact us
at editor@medicalpearlspub.com.

Question 24: What is TRUE about the CT appearance of splenic


hemangiomas?

Choices:
1. It is diagnostic
2. It is variable
3. It is different from that of liver hemangiomas
4. It does not include calcifications

Answer: [2] - It is variable


Explanations:
Splenic hemangiomas on CT can appear just like hepatic hemangiomas, but this
is but one of several non-specific variable appearances.
They may remain hypodense or become hyperdense with contrast.
They may calcify.
They may appear cystic or solid.

Study Topic: Hemangioma


We update our content daily based on user feedback. Please click here to submit
your suggestions or if interested in becoming an author or editor, please contact us
at editor@medicalpearlspub.com.

Question 25: Which gradient is on simultaneously with the production and


recording of the echo?

Choices:
1. Phase
2. Slice selection
3. Frequency
4. None of the above

Answer: [3] - Frequency


Explanations:
The frequency encoding gradient is applied after the 180 degree pulse during the
time that echo is collected.

Study Topic: Magnetic Resonance Imaging (MRI)


We update our content daily based on user feedback. Please click here to submit
your suggestions or if interested in becoming an author or editor, please contact us
at editor@medicalpearlspub.com.

Question 26: Which of the following is NOT true of atelectasis?


Choices:
1. Atelectasis is more common on post-operative days 1 and 2
2. It usually presents with a low-grade fever, dyspnea, and fatigue
3. It is more common in smokers
4. Diagnosis is usually made with a CT scan

Answer: [4] - Diagnosis is usually made with a CT scan


Explanations:
Atelectasis is more common in smokers and that is why patients have to stop
smoking weeks before surgery.
Patients who undergo upper abdominal and thoracic surgery do not take deep
breaths secondary to pain, and thus develop atelectasis.
In overly sedated patients, atelectasis is also more common. It can present with a
low-grade fever and tachypnea.
Atelectasis is usually diagnosed by chest radiograph. The disorder is treated with
incentive spirometry and chest physiotherapy. In the rare patient, bronchoscopy
may be required to help clear all secretions that plug the airways.

Study Topic: Atelectasis


We update our content daily based on user feedback. Please click here to submit
your suggestions or if interested in becoming an author or editor, please contact us
at editor@medicalpearlspub.com.

Question 27: Which of the following is a classic symptom of acute


appendicitis?

Choices:
1. Anorexia
2. Periumbilical pain
3. Right lower quadrant (RLQ) abdominal pain
4. All of the above

Answer: [4] - All of the above


Explanations:
Classic symptoms of acute appendicitis include anorexia and periumbilical pain
Those symptoms are followed by RLQ pain, nausea and vomiting
Only 50% of those with acute appendicitis present with classic symptoms
Appendectomy is the only curative treatment for acute appendicitis

Study Topic: Appendicitis


We update our content daily based on user feedback. Please click here to submit
your suggestions or if interested in becoming an author or editor, please contact us
at editor@medicalpearlspub.com.

Question 28: Which presentation is classic for diverticulosis?


Choices:
1. Bleeding
2. Constipation
3. Infection
4. Obstruction

Answer: [1] - Bleeding


Explanations:
Diverticulosis commonly forms along the vascular penetrating sites of large
intestines.
Since the pouch forms at vessel penetrating sites, heavy bleeding is the most
common presentation.
Sometimes fecal matter accumulates and causes infection.
Diverticulitis can mimic appendicitis if it is located on the right side.

Study Topic: Diverticulosis


We update our content daily based on user feedback. Please click here to submit
your suggestions or if interested in becoming an author or editor, please contact us
at editor@medicalpearlspub.com.

Question 29: The NM RT inadvertently uses a LEAP collimator for a Ga-67


scan. You would expect...

Choices:
1. Septal penetration would increase
2. Sensitivity would decrease
3. Spatial resolution would worsen
4. 1 and 3

Answer: [4] - 1 and 3


Explanations:
Low-energy collimators should not be used for radioactive agents with gamma
peaks greater than 150 Kev.
Since Ga-67 gamma peaks are higher, there will be increased septal penetration.
This will lead to worsening spatial resolution.
It will actually increase sensitivity due to greater counts but decrease specificity
due to greater noise. (It would also increase the number of "non-diagnostic"
studies.)

Study Topic: Gallium Scintigraphy


We update our content daily based on user feedback. Please click here to submit
your suggestions or if interested in becoming an author or editor, please contact us
at editor@medicalpearlspub.com.

Question 30: What is the purpose of performing a cine study on a joint?


Choices:
1. Visualizes motion and joint function
2. Visualizes blood flow
3. Measures muscle tension and strength
4. It is mostly an MRI P.R. tool, creating visually impressive though not clinically
helpful sequences

Answer: [1] - Visualizes motion and joint function


Explanations:
Certain disease processes, such as patellofemoral subluxation, are best seen on
cine motion studies.

Study Topic: Magnetic Resonance Imaging (MRI)


We update our content daily based on user feedback. Please click here to submit
your suggestions or if interested in becoming an author or editor, please contact us
at editor@medicalpearlspub.com.

Question 31: Fertilization most often occurs in which part of the


reproductive system?

Choices:
1. Infundibulum of the uterine tube
2. Fundus of the uterus
3. Ampulla of the uterine tube
4. Myometrium of the uterus

Answer: [3] - Ampulla of the uterine tube


Explanations:
The fallopian tube is divided into infundibulum, ampulla, isthmus, and
interstitial portion (ordered from the ovary to uterus.)
The most common site of human fertilization is the ampulla.
The ampulla is at the intermediate portion of the tube where it dilates as it curves
over the ovary.
The area at greatest risk of catastrophic complication from ectopic implantation
is the interstitial portion.

Study Topic: Embryology, Fertilization


We update our content daily based on user feedback. Please click here to submit
your suggestions or if interested in becoming an author or editor, please contact us
at editor@medicalpearlspub.com.

Question 32: What does the direction of current flow affect in


superconducting magnets?

Choices:
1. Magnetic field strength
2. Magnetic field direction
3. Magnetic field intensity
4. Magnetic field homogeneity

Answer: [2] - Magnetic field direction


Explanations:
According to Faraday's Law, the direction of current flow determines the
direction of the magnetic field.
If you reverse the current flow, you reverse the direction of the magnetic field.

Study Topic: Electromagnetic Induction


We update our content daily based on user feedback. Please click here to submit
your suggestions or if interested in becoming an author or editor, please contact us
at editor@medicalpearlspub.com.

Question 33: Consider a patient obtaining a two-day renogram study, the


first day as a baseline and the second day while on captopril, p.o. On day 2,
GFR in the right kidney, previously within normal range, falls, and peaks
later. What is the most likely etiology of these findings?

Choices:
1. Misadministration of Tc99m-MAG3 on day 2
2. Right sided renovascular hypertension
3. Left sided renovascular hypertension
4. Medication-induced acute renal failure

Answer: [2] - Right sided renovascular hypertension


Explanations:
Captopril is an ACEI, a vasodilator.
Renovascular hypertension is high-blood pressure due to stenosis of the renal
artery.
Captopril renography is a functional test, determining if known hypertension
may be renovascular in origin, and thus treatable (angioplasty, surgery)
The study is positive if a normal baseline study shows dropping GFR,
diminished peak, and delayed peak on the affected side.

Study Topic: Hypertension, Renovascular


We update our content daily based on user feedback. Please click here to submit
your suggestions or if interested in becoming an author or editor, please contact us
at editor@medicalpearlspub.com.

Question 34: What is a sine qua non of a technically good quality first pass
cardiac study?

Choices:
1. Gating
2. An imaging acquisition system with multiple crystals
3. Extending the imaging time
4. Injecting a compact radiopharmaceutical bolus

Answer: [4] - Injecting a compact radiopharmaceutical bolus


Explanations:
The ideal first pass study is based on activity change over time, and in theory is
best with a point bolus. The more compact--rapidly injected--the bolus, the
better.
The other listed choices do not impact on first pass studies.

Study Topic: Nuclear Medicine


We update our content daily based on user feedback. Please click here to submit
your suggestions or if interested in becoming an author or editor, please contact us
at editor@medicalpearlspub.com.

Question 35: A patient with mild dysphagia undergoes endoscopy, which


reveals a squamous cell carcinoma. The most accurate method of T staging is:

Choices:
1. Body CT
2. MRI
3. Endoscopy
4. Endoscopic ultrasound

Answer: [4] - Endoscopic ultrasound


Explanations:
Endoscopic ultrasound is now used to T stage esophageal cancer
Five different layers can be identified throughout the lower esophagus, and three
distinct layers are seen in the upper esophagus
CT or CT/PET can help in N and M staging
In the past, squamous cell was the most common cell type of esophageal
carcinoma in the USA. Now it is adenocarcinoma, though squamous cell
carcinoma remains the most common type of esophageal cancer worldwide

Study Topic: Cancer, Esophageal


We update our content daily based on user feedback. Please click here to submit
your suggestions or if interested in becoming an author or editor, please contact us
at editor@medicalpearlspub.com.

Question 36: You notice a patient must exhale consciously. You deduce that
the lung alveoli have lost their elasticity. You expect the CXR will show areas
of permanent hyperinflation. What is the most probable diagnosis?

Choices:
1. Tuberculosis
2. Asthma
3. Empyema
4. Emphysema

Answer: [4] - Emphysema


Explanations:
Emphysema is a progressive disorder.
Emphysema is a result of long-term bronchial irritation (by air pollution,
cigarette smoking, etc.).
Emphysema prevents normal exhalation because of the loss of alveolar wall
elasticity.
Giving emphysema patients oxygen must be done at a very slow rate because
respiration is controlled by CO2 blood level, which in emphysema cannot be
easily exhaled; increasing the O2 can lead to apnea.

Study Topic: Emphysema


We update our content daily based on user feedback. Please click here to submit
your suggestions or if interested in becoming an author or editor, please contact us
at editor@medicalpearlspub.com.

Question 37: A surgeon wants to know what percentage of respiratory


function will be lost if he performs right lower lobectomy. You quantify the
lung perfusion ROI information as follows: R apical: 150K; R middle: 100K;
R lower: 200K; L upper: 150K; and L lower: 150 K.

Choices:
1. This information is insufficient to answer the surgeon's question
2. The right lower lobe contributes approximately 27% of respiratory capacity
3. The RLL contributes 15% of respiratory capacity
4. The left lung contributes 40% of respiratory capacity

Answer: [2] - The right lower lobe contributes approximately 27% of respiratory
capacity

Explanations:
Total lung perfusion count is 750K. RLL is 200K. So RLL contributes 200/750
= 27% of total lung capacity.
While it is true the left lung contributes 40% (Choice 4), that is not the question
the surgeon needed answering.

Study Topic: Lung, Ventilation Perfusion Scan (VQ Scan)


We update our content daily based on user feedback. Please click here to submit
your suggestions or if interested in becoming an author or editor, please contact us
at editor@medicalpearlspub.com.

Question 38: Soft US findings of breast cancer include all BUT...


Choices:
1. Microlobulations
2. Calcifications
3. Acoustic shadowing
4. Branch pattern

Answer: [3] - Acoustic shadowing


Explanations:
Acoustic shadowing is a hard finding, associated with breast destruction rather
than breast alteration.
Microlobulations, calcificatins, and branch pattern are soft findings, as is Duct
Extension.
Other hard findings besides shadowing include angular margins and spiculation.
Some US findings--like hypoechogenicity and "taller than wide" orientation--are
non-specific, and can be seen in both DCIS and invasive carcinomas.

Study Topic: Cancer, Breast, Ultrasonography


We update our content daily based on user feedback. Please click here to submit
your suggestions or if interested in becoming an author or editor, please contact us
at editor@medicalpearlspub.com.

Question 39: Which complication can result from an improperly placed


nasogastric tube?

Choices:
1. Mediastinitis
2. Bronchiolitis
3. Gastritis
4. Epistaxis

Answer: [1] - Mediastinitis


Explanations:
An improperly placed NG tube can very rarely result in esophageal perforation.
Mediastinitis can follow esophageal perforation.
Bronchiolitis is a viral infection common in children. However, a common
complication of an improperly placed NG tube (in the tracheobronchial tree
rather than the GI tract) is pneumonia if the NG tube is used for feeding.
A properly placed NG tube can cause epistaxis.

Study Topic: Nasogastric Tube


We update our content daily based on user feedback. Please click here to submit
your suggestions or if interested in becoming an author or editor, please contact us
at editor@medicalpearlspub.com.

Question 40: What is not true about radiation exposure in pregnancy?


Choices:
1. Heavy doses of radiation may affect the central nervous system
2. Even high-risk studies such as an abdominal CT scan deliver between 20 and 50
mGy, below the minimum reported threshold of 50 to 100 mGy
3. Radiologic studies can he safe, especially if appropriate abdominal shielding is
used; it leads to abortion 5% of the time
4. If the exposure is greater than 100 cGy, then injury can be expected

Answer: [3] - Radiologic studies can he safe, especially if appropriate abdominal


shielding is used; it leads to abortion 5% of the time

Explanations:
Radiologic studies can be safe, especially if appropriate abdominal shielding is
used.
it leads to abortion less than 1% of the time.

Study Topic: Radiation, Ionizing


We update our content daily based on user feedback. Please click here to submit
your suggestions or if interested in becoming an author or editor, please contact us
at editor@medicalpearlspub.com.

Question 41: Which of the following suggests pyelonephritis rather than an


uncomplicated UTI?

Choices:
1. Fever and chills
2. Frequency, urgency, dysuria
3. Suprapubic pain, low back pain
4. Hematuria

Answer: [1] - Fever and chills


Explanations:
UTI can present with frequency, urgency, dysuria, suprapubic pain, low back
pain, and hematuria
Fever, chills, nausea, vomiting, flank pain, and CVA tenderness are suggestive of
pyelonephritis

Study Topic: Pyelonephritis, Acute


We update our content daily based on user feedback. Please click here to submit
your suggestions or if interested in becoming an author or editor, please contact us
at editor@medicalpearlspub.com.

Question 42: Which of the following arteries does NOT arise from the
superior mesenteric artery?

Choices:
1. Right colic artery
2. Middle colic artery
3. Ileocolic artery
4. Left colic artery

Answer: [4] - Left colic artery


Explanations:
The superior mesenteric artery arises inferior to the origin of the celiac trunk
from the anterior abdominal aorta.
It supplies the pancreas and the lower portion of the duodenum all the way to the
transverse colon.
Branches include the inferior pancreaticoduodenal artery, intestinal arteries,
ileocolic artery, right colic artery, and middle colic artery but not the left colic.
The left colic artery arises from the inferior mesenteric artery.

Study Topic: Anatomy, Abdomen, Arteries, Superior Mesenteric


We update our content daily based on user feedback. Please click here to submit
your suggestions or if interested in becoming an author or editor, please contact us
at editor@medicalpearlspub.com.

Question 43: You notice a hepatocellular carcinoma in a patient WITHOUT


cirrhosis. Compared to hepatocellular carcinoma in those WITH cirrhosis,
which of the following is TRUE?

Choices:
1. More commonly solitary
2. More commonly with scar
3. Both 1 and 2 are true
4. Neither 1 nor 2 is true

Answer: [3] - Both 1 and 2 are true


Explanations:
In those with cirrhosis, only 27% of HCC is solitary. Without cirrhosis, 72% are
solitary.
Scar is seen in only 6% of those with cirrhosis, but half of those without
cirrhosis.

Study Topic: Cancer, Liver


We update our content daily based on user feedback. Please click here to submit
your suggestions or if interested in becoming an author or editor, please contact us
at editor@medicalpearlspub.com.

Question 44: The trachea divides into the left and right mainstem bronchi at
which level?

Choices:
1. T1
2. T4
3. T7
4. C7

Answer: [2] - T4
Explanations:
The carina is usually located at the level of the discs between the T4 and T5
thoracic vertebrae.
The carina divides into right and left mainstem bronchi within the mediastinum.
The right mainstem bronchus is more vertically oriented than the left. For this
reason, aspiration pneumonia and foreign body aspiration is seen more
commonly on the right.
The trachea contains multiple discrete cartilaginous sections (16-20). They are
formed in the anterior two-thirds by hyaline cartilage and in the posterior trachea
by smooth muscle and fibrous tissue.

Study Topic: Anatomy, Thorax, Lungs


We update our content daily based on user feedback. Please click here to submit
your suggestions or if interested in becoming an author or editor, please contact us
at editor@medicalpearlspub.com.

Question 45: Which of the following positions should be avoided for


prolonged periods in individuals with chronic respiratory insufficiency?

Choices:
1. Right lateral decubitus
2. Left lateral decubitus
3. Lying recumbant
4. Sitting in a chair

Answer: [3] - Lying recumbant


Explanations:
Lying recumbant for prolonged periods should be avoided in those with chronic
respiratory insufficiency
The position prevents proper expiratory excursion and is generally
uncomfortable
The best position is sitting in a chair
Sitting promotes proper respiratory excursion, which prevents development of
muscle fatigue and atelectasis

Study Topic: Respiratory Insufficiency


We update our content daily based on user feedback. Please click here to submit
your suggestions or if interested in becoming an author or editor, please contact us
at editor@medicalpearlspub.com.

Question 46: At which location are the pulsations of the posterior tibial
artery best felt?

Choices:
1. Behind the knee
2. Behind the medial malleolus
3. Between the medial and lateral malleoli
4. Behind the lateral malleolus

Answer: [2] - Behind the medial malleolus


Explanations:
The posterior tibial artery is a branch of the popliteal artery that carries blood to
the posterior compartment of the leg and plantar surface of the foot.
The popliteal gives rise to the fibular artery and also gives rise the medial and
lateral plantar arteries.
The posterior tibial artery can be palpated just inferoposterior to the medial
malleoli.
If not palpable, look for physical signs of peripheral arterial disease (cold, hair
loss, bluish skin).

Study Topic: Anatomy, Lower Limb, Calf, Arteries, Posterior Tibial


We update our content daily based on user feedback. Please click here to submit
your suggestions or if interested in becoming an author or editor, please contact us
at editor@medicalpearlspub.com.

Question 47: Which of the following is FALSE about cleft lip?


Choices:
1. Can be associated with cleft palate
2. Occurs equally on upper and lower lip
3. Caused by failure of fusion of maxillary process
4. Can lead to difficulty in breast-feeding

Answer: [2] - Occurs equally on upper and lower lip


Explanations:
Cleft lip is most common on the upper lip. In less than 10% of cases, the lower
lip affected.
The cleft lip may be associated with a cleft palate.
Cleft lip occurs when there is failure of fusion of maxillary process with median
nasal process.
Cleft lips are optimally repaired at 6 to 12 weeks of age.

Study Topic: Cleft Lip


We update our content daily based on user feedback. Please click here to submit
your suggestions or if interested in becoming an author or editor, please contact us
at editor@medicalpearlspub.com.

Question 48: Which of the following is a reliable test to discern


hemidiaphragmatic paralysis?

Choices:
1. MRI of the chest
2. Chest radiograph
3. Fluoroscopy of diaphragm
4. None of the above

Answer: [3] - Fluoroscopy of diaphragm


Explanations:
The fluoroscopic "sniff test" is a reliable investigation to diagnose
hemidiaphragmatic paralysis
The "sniff test" is a real time assessment of diaphragmatic excursion during deep
inhalation and exhalation
CXR and MRI are not valuable in diagnosing hemidiaphragmatic paralysis

Study Topic: Elevated Hemidiaphragm


We update our content daily based on user feedback. Please click here to submit
your suggestions or if interested in becoming an author or editor, please contact us
at editor@medicalpearlspub.com.

Question 49: You notice the QRS complex on a patient's EKG. This
corresponds to what event during the cardiac cycle?

Choices:
1. Atrial depolarization
2. Ventricular depolarization
3. Ventricular repolarization
4. There is no one-to-one correspondence between EKG events and events in the
cardiac cycle

Answer: [2] - Ventricular depolarization


Explanations:
Choice 1 corresponds to the P wave.
Choice 3 corresponds to the T wave.

Study Topic: Rhythm, ECG (EKG)


We update our content daily based on user feedback. Please click here to submit
your suggestions or if interested in becoming an author or editor, please contact us
at editor@medicalpearlspub.com.

Question 50: Using the 5th edition of BIRADS Terminology, mammographic


mass SHAPE descriptors include all BUT...

Choices:
1. Round
2. Oval
3. Macrolobulated
4. Irregular

Answer: [3] - Macrolobulated


Explanations:
In earlier editions of BIRADS Terminology, macrolobulated was a shape choice,
but in the latest, 5th, edition it is subsumed under Oval.
"Irregular" includes all masses that are not round or oval in shape; it carries a
connotation of a suspicious finding.

Study Topic: Breast, Mass


We update our content daily based on user feedback. Please click here to submit
your suggestions or if interested in becoming an author or editor, please contact us
at editor@medicalpearlspub.com.

Question 51: Bone scans are NOT indicated to work up:


Choices:
1. AVN
2. Osteoporosis
3. Osseous metastases
4. Cellulitis

Answer: [2] - Osteoporosis


Explanations:
Although today more commonly worked up with MRI, AVN can be detected with
bone scintigraphy, especially if pinhole collimation is used (e.g., over the hip)
Bone mets are classically worked up with bone scintigraphy
Triple phase bone scans can successfully distinguish cellulitis from underlying
osteomyelitis
Osteoporosis is worked up with bone mineral density (DEXA) scans, including
dual photon absorptiometry (DPA), and dual X-ray absorptiometry (DXA), but
not bone scans. Quantitative CT (QCT) can also be used

Study Topic: Bone, Scan


We update our content daily based on user feedback. Please click here to submit
your suggestions or if interested in becoming an author or editor, please contact us
at editor@medicalpearlspub.com.

Question 52: What condition would have findings of 4mm of nuchal


translucency on an ultrasound of at 12 weeks gestation?

Choices:
1. Brachial cleft cyst
2. Spina bifida
3. Down syndrome
4. Normal development

Answer: [3] - Down syndrome


Explanations:
Ultrasonography before 14 weeks gestation determines gestational age and
detects multiple pregnancies
Nuchal translucency greater than 3 mm at 10 to 14 weeks is associated with
chromosomal abnormalities.
The most common cause is Down syndrome.
Other causes are trisomy 13 (Patau syndrome), trisomy 18 (Edwards syndrome),
and Turner syndrome.

Study Topic: Down Syndrome (Trisomy 21, Down's Syndrome)


We update our content daily based on user feedback. Please click here to submit
your suggestions or if interested in becoming an author or editor, please contact us
at editor@medicalpearlspub.com.

Question 53: Which spinal region is most involved in kyphosis?


Choices:
1. Cervical
2. Thoracic
3. Lumbar
4. Sacral

Answer: [2] - Thoracic


Explanations:
The thoracic spine is most commonly involved with kyphosis.
Kyphotic patients have difficulty with pullover garments.
Severe thoracic kyphosis is often accompanied by compensatory cervical
lordosis.
Kyphosis is often combined with scoliosis, termed kyphoscoliosis.

Study Topic: Kyphosis


We update our content daily based on user feedback. Please click here to submit
your suggestions or if interested in becoming an author or editor, please contact us
at editor@medicalpearlspub.com.

Question 54: Symptoms/Signs of hyperthyroidism include


Choices:
1. exophthalmos
2. tachycardia
3. cold intolerance
4. All of the above

Answer: [2] - tachycardia


Explanations:
Tachycardia, or rapid heart rate, > 100 bpm, is a sign of hyperthyroidism, as is
weight loss with increased appetite, anxiety, increased sweating, menorrhagia,
oily hair, heart palpitations. These all relate to increased metabolism.
Cold intolerance is a sign of HYPOthyroidism. Hyperthyroidism is associated
with heat intolerance (feeling hotter than others in the room).
Exophthalmos--bulging eyes; protruding eyeballs--is associated with Graves
disease, the most common cause of hyperthyroidism. It is NOT a sign of
hyperthyroidism in general.

Study Topic: Thyroid, Hyperthyroidism (T3)


We update our content daily based on user feedback. Please click here to submit
your suggestions or if interested in becoming an author or editor, please contact us
at editor@medicalpearlspub.com.

Question 55: A female with a 45 pack year history presents with weight gain
and a lung mass on chest radiograph. She has hyponatremia and
hyperosmolar urine. What is the most probable diagnosis?

Choices:
1. Congestive heart failure
2. SIADH
3. Acute renal failure
4. Conn syndrome

Answer: [2] - SIADH


Explanations:
SIADH can be caused by a variety of neoplasms but small cell carcinoma of the
lung is the most common
Other malignancies that can cause this are pancreatic, malignant lymphoma, and
thymus gland tumors
The excess ADH causes water retention and concentration of the urine
Other nonmalignant causes include infection, sarcoidosis, medications, and
subarachnoid hemorrhage

Study Topic: Syndrome Of Inappropriate Antidiuretic Hormone Secretion


(SIADH)
We update our content daily based on user feedback. Please click here to submit
your suggestions or if interested in becoming an author or editor, please contact us
at editor@medicalpearlspub.com.

Question 56: A young male who is a glass blower presents to you with a
bulging mass on the upper chest. A diagnosis of a soft non-tender crepitant
swelling is made. Which radiograph would be most diagnostic?

Choices:
1. PA (posteroanterior) chest radiograph
2. Lateral chest radiograph
3. Oblique radiograph tangential to the defect
4. Ribs views

Answer: [3] - Oblique radiograph tangential to the defect


Explanations:
The history of glass blowing indicates increased intrathoracic pressure. The
history, in addition to the physical exam findings, suggests a lung hernia. Lung
hernias can result from defects in the diaphragm or intercostal muscles.
They are generally seen in COPD patients, but can also be an occupational
hazard as in this case. About 1/3 are noted in the supraclavicular area. The hernia
may present as a non-tender crepitant swelling which enlarges on forced
expiration.
The intercostal variety is the most common (60-80%) and occurs when the
intercostal muscle development is incomplete.
Pneumonocele, or hernia of the lung, is best diagnosed on an oblique radiograph
tangential to the defect, especially during a Valsalva maneuver. Surgery is usually
necessary and may require a prosthetic mesh to close the defect.

Study Topic: Hernia, Lung


We update our content daily based on user feedback. Please click here to submit
your suggestions or if interested in becoming an author or editor, please contact us
at editor@medicalpearlspub.com.

Question 57: Dose calibrator linearity is checked:


Choices:
1. On installation
2. Monthly
3. Quarterly
4. 1 and 3

Answer: [4] - 1 and 3


Explanations:
Testing linearity means testing the accuracy of measurement of the same
radionuclide over a range of activities.
This should be done within a 24 hour period.
It should be performed at the time of installation and quarterly thereafter.
NRC requires records of this be kept for 3 years.

Study Topic: Quality, Control Metrics


We update our content daily based on user feedback. Please click here to submit
your suggestions or if interested in becoming an author or editor, please contact us
at editor@medicalpearlspub.com.

Question 58: Loss of control of the external urethral sphincter would result
from damage to which structure?

Choices:
1. The vagus nerve
2. The pudendal nerve
3. The inferior hypogastric plexus
4. The sympathetic trunk

Answer: [2] - The pudendal nerve


Explanations:
The vagus nerve has no function related to micturition
Voluntary motor innervation to the external urethral sphincter is via the pudendal
nerve
The sympathetic trunk and inferior hypogastric plexus are involved in bladder
function

Study Topic: Anatomy, Pelvis, Nerves, Pudendal


We update our content daily based on user feedback. Please click here to submit
your suggestions or if interested in becoming an author or editor, please contact us
at editor@medicalpearlspub.com.

Question 59: Which of the following distances should be of concern with


respect to "geometric unsharpness"?

Choices:
1. Object to image
2. Source to object
3. Source to image
4. All of the above

Answer: [4] - All of the above


Explanations:
As OID increases, so does magnification. OID is directly proportional to
magnification,
As focal-object distance and SID decrease, there is a corresponding increase in
magnification. FOD and SID are both inversely proportional to magnification.
The more magnified an image, the more geometrically unsharp the image.

Study Topic: Radiology, Image Production and Evaluation


We update our content daily based on user feedback. Please click here to submit
your suggestions or if interested in becoming an author or editor, please contact us
at editor@medicalpearlspub.com.

Question 60: The ratio of the lead strip height to the distance between lead
strips is referred to as the:

Choices:
1. Grid ratio
2. Grid length ratio
3. Lead ratio
4. Grid lead ratio

Answer: [1] - Grid ratio


Explanations:
This is the definition of the grid ratio

Study Topic: Grid Ratio


We update our content daily based on user feedback. Please click here to submit
your suggestions or if interested in becoming an author or editor, please contact us
at editor@medicalpearlspub.com.

Question 61: What is an emetic?


Choices:
1. An agent that induces vomiting
2. An agent that stimulates defecation
3. An agent that promotes voiding
4. An agent that suppresses cough

Answer: [1] - An agent that induces vomiting


Explanations:
An emetic is an agent that induces vomiting. A classic emetic is syrup of ipecac.
Cathartics stimulate defecation.
Diuretics promote voiding.
Antitussive suppress coughs.

Study Topic: Vomiting


We update our content daily based on user feedback. Please click here to submit
your suggestions or if interested in becoming an author or editor, please contact us
at editor@medicalpearlspub.com.

Question 62: Which of the following is NOT an advantage of arteriography


over radionuclide RBC scans?

Choices:
1. More specific
2. Has therapeutic potential
3. Increased sensitivity
4. Faster

Answer: [3] - Increased sensitivity


Explanations:
Angiography is sometimes used as the initial study in patients with massive
lower GI bleed. Individuals who require more than four units of blood with the
first 2 hours of admission and have a systolic blood pressure less than 100
mmHg need more aggressive measures.
However, angiography also entails the use of iodinated contrast that can be
harmful in the presence of hypotension.
One of the main advantages of conventional angiography is the ability to perform
selective arterial infusion of vasoconstrictive drugs, or embolization of
particulate matter into the bleeding artery.
Selective angiography can detect bleeds at a rate of 1-2 cc/min while
radionuclide exams can detect a bleeding rate of 0.1-0.2 cc/min. The nuclear
study is also more sensitive because with intermittent bleeds you can image out
to 24 hours on the single radionuclide dose.

Study Topic: Gastrointestinal Bleeding


We update our content daily based on user feedback. Please click here to submit
your suggestions or if interested in becoming an author or editor, please contact us
at editor@medicalpearlspub.com.

Question 63: The NM RT is calculating the LVEF on a patient with an


enlarged left atrium. You notice that as a consequence, the LV ROI includes
some of the LA. This will cause the LVEF calculation to be artifactually...

Choices:
1. increased
2. decreased
3. increased or decreased
4. cannot determine from information given

Answer: [2] - decreased


Explanations:
If you include more in the ROI than is appropriate, you are increasing the value
of both the LVED and LVES.
In the numerator, this addition may subtract out, but in the denominator, you will
have a number (count) larger than otherwise.
Dividing by a larger number will artificially lower the total number. So choice 2
is correct.

Study Topic: Left Ventricular Ejection Fraction


We update our content daily based on user feedback. Please click here to submit
your suggestions or if interested in becoming an author or editor, please contact us
at editor@medicalpearlspub.com.

Question 64: What disease process of the ear can involve the malleus?
Choices:
1. Otosclerosis
2. Cholesteatoma
3. Mastoiditis
4. Meniere disease

Answer: [2] - Cholesteatoma


Explanations:
Because the malleus is directly connected to the tympanic membrane, it is
unlikely that it will be the cause of hearing loss.
In case of atticoantral disease, which is a severe inflammatory disorder of the
middle ear caused by excessive skin growth the middle ear ossicles can be
affected.
Cholesteatoma is a very fast and destructive lesion that consist of growth of
keratinized squamous epithelium. When it grows it can involve the ossicles, and
lead to hearing loss.
During surgery to remove the cholesteatoma, the ossicles may need to be
removed. Most patients require a second procedure for reconstruction of the
middle ear.

Study Topic: Cholesteatoma


We update our content daily based on user feedback. Please click here to submit
your suggestions or if interested in becoming an author or editor, please contact us
at editor@medicalpearlspub.com.

Question 65: Which is TRUE concerning family members and ancillary


personnel who enter the MRI magnet room with the patient?

Choices:
1. They need not be screened; they are not entering the MR bore
2. They can check on the patient in the room but cannot stay in the room during the
procedure
3. They must be screened identically to patients having an MRI themselves, as the
entire room is within the critical zone of the magnet
4. They must wear lead aprons and safety goggles at all times

Answer: [3] - They must be screened identically to patients having an MRI


themselves, as the entire room is within the critical zone of the magnet

Explanations:
Per ACR White Paper on MR safety guidelines, there are four zones of interest.
Zone IV is synonymous with the MR scanner magnet room itself.
Zone IV, by definition, will always be located within Zone III, as it is the MR
magnet and its associated magnetic field that generates the existence of Zone III.
Zone III is the region in which free access by unscreened non-MR personnel can
result in serious injury or death as a result of interactions between the
individuals and the MR scanner's particular environment.
All access to Zone III is to be strictly restricted. MR technologists are charged
with ensuring that this MR safe practice guideline is strictly adhered to for the
safety of the patients and other non-MR personnel.

Study Topic: Magnetic Resonance Imaging (MRI)


We update our content daily based on user feedback. Please click here to submit
your suggestions or if interested in becoming an author or editor, please contact us
at editor@medicalpearlspub.com.

Question 66: Which is correct about PET imaging for adrenal metastases?
Choices:
1. Sensitivity is > 90%
2. Specificity is > 90%
3. Both 1 and 2 are true
4. Neither 1 nor 2 are true

Answer: [3] - Both 1 and 2 are true


Explanations:
Literature on PET in lung cancer shows adrenal metastases are detected (and
distinguished from non-metastatic adrenal masses) with a sensitivity ranging
from 93-100% and a specificity ranging from 90-94%.
You would only expect PET to highlight the metastatic disease of those
neoplasms in which PET is sensitive in detecting the primary. The most common
primaries that metastasize to adrenal are breast, lung, melanoma, and renal cell
carcinoma. PET is excellent at detecting breast, lung, and melanoma.
Of interest, PET has a lower sensitivity in detection of renal cell carcinoma,
NOT because the primary malignancy does not take up the tracer, but because
the tracer is ALSO taken up normally by the normal kidney. As a result, adrenal
metastases from RCC are EASIER to detect than the primary itself with PET
imaging.

Study Topic: Cancer, Metastasis, Adrenal


We update our content daily based on user feedback. Please click here to submit
your suggestions or if interested in becoming an author or editor, please contact us
at editor@medicalpearlspub.com.

Question 67: You are performing a one day two phase (rest/stress) Tc99m
Sestamibi myocardial perfusion SPECT scan. Which statement is true?

Choices:
1. The Sestamibi doses for both stress and rest must be identical
2. The higher dose of sestamibi should always be the stress dose
3. The second dose should always be higher than the first dose
4. You should always get the rest images before the stress images

Answer: [3] - The second dose should always be higher than the first dose
Explanations:
If you were doing a two-day protocol, you would give identical doses of the
radiopharmaceutical, but this is not recommended for a one-day protocol.
To get the study done in one day with a radiopharmaceutical with a 6 hour halflife, you have to give two different doses, and the higher dose must be given
second.
There are two advantages to doing the stress study first. 1: If the stress study is
normal, you don't need a rest study.
2: if the stress study has defects, you can see if they "fill in" on rest, whereas if
the rest study is normal, it is harder to see a superimposed defect on stress
(though with a larger dose it IS possible.)

Study Topic: Myocardial Perfusion Scan


We update our content daily based on user feedback. Please click here to submit
your suggestions or if interested in becoming an author or editor, please contact us
at editor@medicalpearlspub.com.

Question 68: Which radiographic signs is associated with duodenal


obstruction?

Choices:
1. Bird's beak sign
2. Hampton's hump
3. Double bubble sign
4. Septal lines sign

Answer: [3] - Double bubble sign


Explanations:
Congenital duodenal obstruction can be caused by several different anomalies,
including duodenal atresia, duodenal stenosis, and malrotation of the intestine.
Infants with Down syndrome are at increased risk.
Symptoms include projectile vomiting after the first few feedings.
Polyhydramnios may also be present during pregnancy and is caused by a failure
of absorption of amniotic fluid in the distal intestine. Diagnosis is usually made
radiographically.
Plain radiographs show the characteristic "double bubble" sign: one large bubble
in the stomach with a smaller adjacent bubble that represents the duodenum. If
atresia is present, no abdominal gas is seen in the distal bowel. However, if
stenosis is present, a small amount of gas may be present.
The diagnosis is often made on fetal ultrasound. In the neonate, a barium upper
gastrointestinal series should clarify etiology. Treatment involves nasogastric
suction to decompress the stomach and surgery to correct the obstructing lesion.

Study Topic: Duodenal Atresia And Stenosis


We update our content daily based on user feedback. Please click here to submit
your suggestions or if interested in becoming an author or editor, please contact us
at editor@medicalpearlspub.com.

Question 69: If a kit contains 100 mCi in 25 cc at 8 AM of a Tc99m


radiopharmaceutical, how much should be withdrawn at 2 PM the same day to
fill a 4 mCi dose?

Choices:
1. 1 cc
2. 2 cc
3. 4 cc
4. Not enough information

Answer: [2] - 2 cc
Explanations:
There are two problems here. Problem 1 is what is left at 2 PM after decay.
Problem 2 is what volume corresponds to what dose.
There are 6 hours between 8 AM and 2 PM. This is one half-life for Tc99m. So
by physical decay, the 100 mCi is now 50 mCi.
Nothing has been done, so there is still 25 cc, or 2 mCi/cc.
You want 4 mCi. At 2 mCi/cc, you need 2 cc, or Choice 2. If you forgot to
consider physical decay, you might have chosen Choice 1. If you thought the
half-life of Tc99m was 3 hours instead of 6, you might have chosen choice 3.

Study Topic: Nuclear Medicine


We update our content daily based on user feedback. Please click here to submit
your suggestions or if interested in becoming an author or editor, please contact us
at editor@medicalpearlspub.com.

Question 70: With a Baker cyst in the popliteal fossa, you would expect what
condition?

Choices:
1. Rheumatoid arthritis
2. Osteoarthrosis
3. Deep vein thrombosis
4. Sarcoidosis

Answer: [2] - Osteoarthrosis


Explanations:
In most people who have a Baker cyst, otherwise known as a popliteal cyst,
osteoarthrosis is a commonly associated disorder. These are not true cysts, as
they typically openly communicate with the synovial joint.
Choice 1 is not completely incorrect, but less likely. Other disorders where one
may find Baker's cyst include gout, rheumatoid arthritis, Reiter syndrome and
lupus
Choice 4 is not completely incorrect, but less likely. Systemic diseases
associated with Baker's cyst include hemodialysis complications, infection,
hypothyroidism, and sarcoidosis.
The most important differential diagnosis when a cyst is palpated in the popliteal
fossa is a popliteal artery aneurysm, which should be pulsatile. Color flow
Doppler US easily distinguishes.

Study Topic: Cyst, Baker


We update our content daily based on user feedback. Please click here to submit
your suggestions or if interested in becoming an author or editor, please contact us
at editor@medicalpearlspub.com.

Question 71: In the ACRIN 6666 trials on breast MRI, what percentage of
eligible women refused the offered (free) breast MRI?

Choices:
1. 5%
2. 17%
3. 42%
4. 60%

Answer: [3] - 42%


Explanations:
42% refused the free offered MRI
Reasons included refusal of injection (6%), possibility of additional
testing/biopsy as a result (5%), scheduling constraints (4%), and need to travel
(2%)

Study Topic: Magnetic Resonance Imaging (MRI)


We update our content daily based on user feedback. Please click here to submit
your suggestions or if interested in becoming an author or editor, please contact us
at editor@medicalpearlspub.com.

Question 72: Regarding BUS descriptors for calcifications, if present, they


can be described as...

Choices:
1. Macrocalcifications (> 0.5 mm)
2. Microcalcifications within a mass
3. Microcalcifications without a mass
4. All of the above

Answer: [4] - All of the above


Explanations:
All choices are acceptable descriptors of US demonstrated calcifications.
In the past, US was not that of any value in detection/assessment of breast
calcifications. This is no longer the case.

Study Topic: Breast, Imaging, Reporting and Data System (BI RADS)
We update our content daily based on user feedback. Please click here to submit
your suggestions or if interested in becoming an author or editor, please contact us
at editor@medicalpearlspub.com.

Question 73: About "focal asymmetry," which statement is FALSE?


Choices:
1. Similar appearance on two views
2. Convex outward
3. Usually interspersed with fat
4. Similar to global asymmetry, but smaller (involving less than one quadrant)

Answer: [2] - Convex outward


Explanations:
Focal asymmetry, distinct from a MASS lesion, is conCAVE outward.

Study Topic: Breast, Imaging, Reporting and Data System (BI RADS)
We update our content daily based on user feedback. Please click here to submit
your suggestions or if interested in becoming an author or editor, please contact us
at editor@medicalpearlspub.com.

Question 74: Who is most likely to have hyperparathyroidism?


Choices:
1. Women in their childbearing years
2. Postmenopausal women
3. Elderly men
4. School age girls

Answer: [2] - Postmenopausal women


Explanations:
Postmenopausal women are most likely to have hyperparathyroidism

Study Topic: Hyperparathyroidism


We update our content daily based on user feedback. Please click here to submit
your suggestions or if interested in becoming an author or editor, please contact us
at editor@medicalpearlspub.com.

Question 75: The parenchyma of the testis is composed of lobules separated


by septae composed of what anatomic structure?

Choices:
1. Visceral layer, tunica vaginalis
2. Parietal layer, tunica vaginalis
3. Tunica albuginea
4. Rete testis

Answer: [3] - Tunica albuginea


Explanations:
The tunica albuginea divides the testis into lobules by extending septae into the
testis parenchyma
The visceral layer of the tunica vaginalis surrounds the testis, epididymis, and
spermatic cord except where the epididymis and testis are fused to the scrotal
wall
The inside of the scrotum is lined by the parietal layer of the tunica vaginalis
The rete testis connects testis with epididymis

Study Topic: Anatomy, Pelvis, Testes


We update our content daily based on user feedback. Please click here to submit
your suggestions or if interested in becoming an author or editor, please contact us
at editor@medicalpearlspub.com.

Question 76: Which is not used as treatment of pulmonary embolism?


Choices:
1. Inferior vena cava filter
2. Thrombolytics
3. Heparin
4. Aspirin

Answer: [4] - Aspirin


Explanations:
Heparin and thrombolytics are routinely used in the treatment of pulmonary
embolism.
Heparin prevents thrombus propagation and inhibits serotonin release.
Thrombolytics can be utilized to lyse large pulmonary emboli.
IVC filter placement is indicated for complications of anticoagulation,
contraindications to anticoagulation, or failure of anticoagulation to prevent
recurrent thrombi.
Aspirin has no place in the treatment of pulmonary embolism

Study Topic: Embolism, Pulmonary


We update our content daily based on user feedback. Please click here to submit
your suggestions or if interested in becoming an author or editor, please contact us
at editor@medicalpearlspub.com.

Question 77: Select the true statement about hydatid cysts.


Choices:
1. They are caused by Entamoeba histolytica
2. Diagnosis is confirmed by aspiration
3. The most commonly affected organs are the liver and lungs
4. Treatment with one antiparasitic drug is recommended

Answer: [3] - The most commonly affected organs are the liver and lungs
Explanations:
The infection is caused by echinococcosis. Entamoeba histolytica leads to
amebic cysts.
Aspiration of a cyst with spillage can result in anaphylaxis, (or so it is taught,
though in most cases inadvertent percutaneous drainage does not lead to
anaphylaxis).
Most patients are asymptomatic unless it causes mass effect.
Treatment is with albendazole and mebendazole.

Study Topic: Echinococcus Granulosus (Hydatid Cysts, Echinococcosis)


We update our content daily based on user feedback. Please click here to submit
your suggestions or if interested in becoming an author or editor, please contact us
at editor@medicalpearlspub.com.

Question 78: Which of the following statements about the pericardium is not
true?

Choices:
1. The majority of pericardial cysts are located in the right costophrenic angle
2. Pericardial cysts are generally asymptomatic
3. Pericardial cysts rarely communicate with the pericardium
4. Pericardial cysts can be observed

Answer: [1] - The majority of pericardial cysts are located in the right
costophrenic angle

Explanations:
Most pericardial cysts are seen on a chest radiograph and are usually
asymptomatic.
They contain yellow clear fluid and usually do not communicate with the
pericardium.
Most are located in the right cardiophrenic space.
The fluid can be drained under CT guidance, but most can be observed.

Study Topic: Cyst, Pericardial


We update our content daily based on user feedback. Please click here to submit
your suggestions or if interested in becoming an author or editor, please contact us
at editor@medicalpearlspub.com.

Question 79: Two patients, 1 and 2, are receiving myocardial perfusion PET
exams. Both 1 and 2 are of similar height, but A weighs only 100 pounds while
B weighs 200 lb. There will be more true coincidence detection with patient...

Choices:
1. A
2. B
3. It will be the same with both patients; it will just take longer with patient B
4. True coincidence detection is a function of camera arrangement, not a function of
the patient

Answer: [1] - A
Explanations:
The thinner patient, A, will have less attenuation of annihilation photons, and
thus more coincidence detection per unit time.

Study Topic: PET Scanning


We update our content daily based on user feedback. Please click here to submit
your suggestions or if interested in becoming an author or editor, please contact us
at editor@medicalpearlspub.com.

Question 80: Select the true statement about mesenteric venous occlusion.
Choices:
1. Bowel resection is rarely required
2. Symptoms evolve over a week to 10 days
3. The etiology is usually clear
4. Treatment is with vasodilators

Answer: [2] - Symptoms evolve over a week to 10 days


Explanations:
Mesenteric venous occlusion results in bowel ischemia requiring resection
The etiologies are numerous but the cause is only found in about half
The symptoms progress over 7 to 10 days
Vasodilators are ineffective

Study Topic: Mesenteric Venous Thrombosis


We update our content daily based on user feedback. Please click here to submit
your suggestions or if interested in becoming an author or editor, please contact us
at editor@medicalpearlspub.com.

Question 81: The mass descriptor unique to BUS is...


Choices:
1. Shape
2. Size
3. Orientation
4. Margin

Answer: [3] - Orientation


Explanations:
Mammography can describe lesion shape, size, and margin characteristics, but
lesion orientation is a unique US parameter.

Study Topic: Breast, Imaging, Reporting and Data System (BI RADS)
We update our content daily based on user feedback. Please click here to submit
your suggestions or if interested in becoming an author or editor, please contact us
at editor@medicalpearlspub.com.

Question 82: Brain abscesses in North America are most commonly seen in
what population?

Choices:
1. Individuals involved with head trauma
2. Those with abscesses elsewhere in the body
3. Those with a CD4 count of less than 300
4. Individuals who have surgery on the brain

Answer: [3] - Those with a CD4 count of less than 300


Explanations:
Brain abscesses are rare in the general population. Immunocompromised patients
have the highest incidence of fungal or protozoal organisms causing this
condition.
Brain abscess can develop following head trauma in a small percentage patients.
Headache is the most common presenting sign. Fever or neurologic deficits are
sometimes seen.
Sudden worsening of a preexisting headache accompanied with signs of
meningismus may be indicative of rupture of abscess into the ventricular space.

Study Topic: Abscess, Brain


We update our content daily based on user feedback. Please click here to submit
your suggestions or if interested in becoming an author or editor, please contact us
at editor@medicalpearlspub.com.

Question 83: Why must you give two doses of sestamibi to perform a
stress/rest myocardial perfusion scan when you need give only one dose of
thallous chloride?

Choices:
1. The different half-lives of Tl-201 and Tc99m
2. The too rapid washout of sestamibi
3. The fact sestamibi doesn't redistribute like thallium does
4. It would always be better to give two doses, but thallium is too expensive to do so

Answer: [3] - The fact sestamibi doesn't redistribute like thallium does
Explanations:
Thallium redistributes, so the initial post-stress exam informs as to myocardial
perfusion at stress and later, without a second dose, re-imaging at rest gives rest
data.
Sestamibi does not redistribute. It is fixed in the myocardial cells. So a second
dose is needed for studying both rest and stress.
The physical half-lives of thallium and technetium have nothing to do with it.

Study Topic: Myocardial Perfusion Scan


We update our content daily based on user feedback. Please click here to submit
your suggestions or if interested in becoming an author or editor, please contact us
at editor@medicalpearlspub.com.

Question 84: Where are benign gastric ulcers most commonly located?
Choices:
1. Greater curvature
2. Lesser curvature
3. Fundus
4. Pylorus

Answer: [2] - Lesser curvature


Explanations:
The majority of benign ulcers are located in the lesser curvature, especially in the
region of the incisura.
Benign ulcers may also occur on the posterior gastric wall.
Ulcers less than 1 cm in diameter are usually benign.
Although infrequently performed today, a technically optimal double-contrast
upper GI series has a very high accuracy in determining whether or not a gastric
ulcer is benign or malignant.

Study Topic: Ulcer, Gastric


We update our content daily based on user feedback. Please click here to submit
your suggestions or if interested in becoming an author or editor, please contact us
at editor@medicalpearlspub.com.

Question 85: A male patient complains of leg weakness for 4 days. He


reports increasing weakness with incontinence and loss of sensation from the
waist down. He has no significant past medical history and has not travelled
recently. The strength of the lower extremities is 3/5 and the sensory level is at
the umbilicus. Mental status, cerebellar exam, and reflexes are normal. Which
of the following tests is not necessary?

Choices:
1. MRI of the spine
2. Electromyography
3. Lumbar puncture
4. Antinuclear antibodies

Answer: [2] - Electromyography


Explanations:
The most likely diagnosis is myelopathy
The lack of pain and rapidity of onset make compression of the cord unlikely
MRI will rule out structural lesion and non compressive myelopathy is caused by
either spinal cord infarction, vasculitis, sarcoidosis, infections (especially viral),
or demyelinating disease
Electromyography would not be helpful for these diagnoses

Study Topic: Myelopathy


We update our content daily based on user feedback. Please click here to submit
your suggestions or if interested in becoming an author or editor, please contact us
at editor@medicalpearlspub.com.

Question 86: Today many radioiodine therapy administrations are done on


an outpatient basis. In the past, the initial therapy was done in an inpatient
context to minimize radiation to others, but regulations have relaxed over time.
Nonetheless, in keeping with ALARA, what recommendations should you give
to patients given large doses of I-131?

Choices:
1. Drink lots of fluid and void frequently
2. Minimize close contact with friends, family
3. Flush toilet at least twice after each use
4. All of the above

Answer: [4] - All of the above


Explanations:
All of these common sense recommendations limit radiation exposure to others,
who gain no benefit from the radiation.
ALARA means "as low as reasonably achievable." All of the above
recommendations are reasonable to comply with.

Study Topic: ALARA (As low as reasonably achievable)


We update our content daily based on user feedback. Please click here to submit
your suggestions or if interested in becoming an author or editor, please contact us
at editor@medicalpearlspub.com.

Question 87: What is the most sensitive technique for detection of a lower
gastrointestinal bleed?

Choices:
1. Colonoscopy
2. Radionuclide study
3. Conventional angiography
4. Computed tomography (CT)

Answer: [2] - Radionuclide study


Explanations:
In order to identify lower gastrointestinal bleeding, the patient must be actively
bleeding at the time.
There are two radionuclide studies to detect gastrointestinal bleeding. Tc99m
labeled sulfur colloid can detect bleeds as slow as 0.05-0.1 cc/min. However, the
bleed must be active at the time of administration, since sulfur colloid is rapidly
removed from the circulation by the liver and spleen. For this reason, it is also
less helpful for bleeds in the upper abdomen.
Tc99m-labeled red blood cells can detect bleeds at a rate as low as 0.1-0.2
cc/min, but if the bleed is not active at the time of study, the exam may be
repeated with the injected dose for up to 24 hours. Conventional angiography
can only detect faster bleeding, at a rate of about 1-2 cc/min.
In most cases of lower gastrointestinal bleeding, the bleeding usually stops
spontaneously. Re-bleeding is common in many cases of lower gastrointestinal
bleeds.

Study Topic: Gastrointestinal Bleeding, Lower


We update our content daily based on user feedback. Please click here to submit
your suggestions or if interested in becoming an author or editor, please contact us
at editor@medicalpearlspub.com.

Question 88: What is lymphoscintigraphy used for?


Choices:
1. It can map lymphatic channels
2. It can show how malignant tumors drain--to which lymph nodes
3. It can identify the sentinel node
4. All are true

Answer: [4] - All are true


Explanations:
All three statements are correct.
Lymphoscintigraphy is typically performed with a radiocolloid injected
peritumorally. Dynamic and delayed imaging can be performed.

Study Topic: Lymphoscintigraphy


We update our content daily based on user feedback. Please click here to submit
your suggestions or if interested in becoming an author or editor, please contact us
at editor@medicalpearlspub.com.

Question 89: Which of the following forms the largest part of the
sternocostal surface of the heart?

Choices:
1. Right atrium
2. Left ventricle
3. Left atrium
4. Right ventricle

Answer: [4] - Right ventricle


Explanations:
The right ventricle lies behind the sternum and forms the largest part of the
sternocostal surface and inferior border of the heart.
The right ventricle is triangular, extending from the right atrium to near the apex.
Its anterosuperior surface is convex.
Its inferior surface is flattened and rests upon the diaphragm.

Study Topic: Anatomy, Thorax, Heart


We update our content daily based on user feedback. Please click here to submit
your suggestions or if interested in becoming an author or editor, please contact us
at editor@medicalpearlspub.com.

Question 90: Which of the following statements about biliary cystadenomas


is FALSE?

Choices:
1. Pre-malignant
2. Typically multilocular
3. No gender predilection
4. Can contain mucinous fluid

Answer: [3] - No gender predilection


Explanations:
"No gender predilection" is false. There is a clear female predilection for these
lesions.
These lesions are viewed as pre-malignant, at risk for degenerating into biliary
cystadenocarcinomas. As such, they are typically resected prophylactically.
Histologic assessment of these lesions can find either serous or mucinous fluid,
so "can contain mucinous fluid" is true.

Study Topic: Cancer, Biliary Tract


We update our content daily based on user feedback. Please click here to submit
your suggestions or if interested in becoming an author or editor, please contact us
at editor@medicalpearlspub.com.

Question 91: Certain patients who are immobile can develop heterotopic
ossification. Which of the following conditions is least likely to result in this?

Choices:
1. Poliomyelitis
2. Spinal cord injury
3. Guillain-Barre syndrome
4. Rheumatoid arthritis

Answer: [4] - Rheumatoid arthritis


Explanations:
Patient conditions most likely to develop heterotopic ossification include
tetanus, burns, paralysis for mechanical ventilation, Guillain-Barre syndrome,
SCI, poliomyelitis, and ALS
RA is less likely to have this complication

Study Topic: Heterotopic Ossification


We update our content daily based on user feedback. Please click here to submit
your suggestions or if interested in becoming an author or editor, please contact us
at editor@medicalpearlspub.com.

Question 92: What unit of measurement is the roentgen?


Choices:
1. Radiation dose
2. Biologic equivalent
3. Dose equivalent
4. Ionization in air

Answer: [4] - Ionization in air


Explanations:
Radiation can be measured various in various ways.
The Rad stands for radiation absorbed dose and measures energy deposited from
ionization in any kind of absorber.
This is different from measuring the radiation the absorber is exposed to. That
unit of exposure, the roentgen, expresses the quantity of ionization in air.
Both the radiation exposure and the radiation absorption are different than the
biological consequence. That unit is measured in dose equivalents, Rem
(radiation equivalent man).

Study Topic: Radiation, Ionizing Exposure, Medical Imaging


We update our content daily based on user feedback. Please click here to submit
your suggestions or if interested in becoming an author or editor, please contact us
at editor@medicalpearlspub.com.

Question 93: Which are of the brain in patients with migraines has been
found to have changes on MRI?

Choices:
1. Periaqueductal gray matter
2. Cerebellum
3. Frontal lobes
4. All of the above

Answer: [4] - All of the above


Explanations:
Migraine causes progressive brain disease especially in the cerebellum,
brainstem, and frontal lobes
Increasing attack frequency correlates with increased risk of subcortical white
matter hyperdensities
The periaqueductal gray area can show increased deposition of non heme iron
Cerebellar infarcts are often present

Study Topic: Headache, Migraine


We update our content daily based on user feedback. Please click here to submit
your suggestions or if interested in becoming an author or editor, please contact us
at editor@medicalpearlspub.com.

Question 94: What nerve is compressed in carpal tunnel syndrome?


Choices:
1. Ulnar
2. Radial
3. Median
4. Brachial

Answer: [3] - Median


Explanations:
The median nerve travels through the carpal tunnel and is compressed in patients
with CTS

Study Topic: Carpal Tunnel Syndrome


We update our content daily based on user feedback. Please click here to submit
your suggestions or if interested in becoming an author or editor, please contact us
at editor@medicalpearlspub.com.

Question 95: You are performing a quantitative lung perfusion scan. You
have drawn ROIs around both right and left lungs from posterior images.
Right ROI was 150K. Left ROI was 200K. What is the percentage perfusion of
the right lung?

Choices:
1. 43%
2. 57%
3. You cannot say. You need anterior image quantification as well
4. This is of no clinical interest. There is never any need to quantify lung perfusion
in this fashion

Answer: [3] - You cannot say. You need anterior image quantification as well
Explanations:
Lung perfusion fractions are determined by calculating geometric means using
both anterior and posterior projection data acquisition. The geometric mean of
two values is the square root of the product of the values.
To use only anterior or posterior images would be misleading. Anteriorly, the
heart is on the left, so there is more lung posteriorly on the left.
Lung perfusion quantification is very helpful is determining if the patient is a
candidate for bullectomy or pneumonectomy. So Choice 4 is incorrect.

Study Topic: Lung, Ventilation Perfusion Scan (VQ Scan)


We update our content daily based on user feedback. Please click here to submit
your suggestions or if interested in becoming an author or editor, please contact us
at editor@medicalpearlspub.com.

Question 96: You are given the following facts: An MAA kit contains 950K
particles per ml. The kit contains 50 mCi Tc99m in 5 ml. A 4 mCi dose is drawn
up. How many particles does it contain?

Choices:
1. 80,000
2. 380,000
3. 400,000
4. 600,000

Answer: [2] - 380,000


Explanations:
50 mCi in 5 ml is 10 mCi/ml.
4 mCi is desires, so you want a volume of 0.4 ml.
There are 950K particles per ml. So in 0.4 ml there are 380K particles.
A normal dose is 100-600K, so this is an acceptable dose.

Study Topic: Nuclear Medicine


We update our content daily based on user feedback. Please click here to submit
your suggestions or if interested in becoming an author or editor, please contact us
at editor@medicalpearlspub.com.

Question 97: If you create an object-image difference of 6 inches, what


happens to image contrast?

Choices:
1. Nothing; an OID affects magnification, not contrast
2. Contrast increases
3. Contrast decreases
4. The effect of OID increase on contrast varies based on other factors

Answer: [2] - Contrast increases


Explanations:
An OID > 0 is an air-gap.
Air gaps decrease scatter radiation; that is, less radiation scatter from the body
reaches the image.
In effect, then, the increased OID has the same action as a low ratio grid. Grids
increase image contrast.

Study Topic: Radiology, Image Production and Evaluation


We update our content daily based on user feedback. Please click here to submit
your suggestions or if interested in becoming an author or editor, please contact us
at editor@medicalpearlspub.com.

Question 98: Which is a feature of familial polyposis coli (FPC)?


Choices:
1. Polyps are present at birth
2. Colorectal cancers do not occurs in all patients with FPC
3. Surgery involves segmental resection to remove the polyps
4. Sulindac can reduce the number and size of polyps

Answer: [4] - Sulindac can reduce the number and size of polyps
Explanations:
Drugs like celecoxib and sulindac have been used successfully to reduce the
number and size of polyps.
Screening with upper and lower endoscopy every 1-3 years is recommended to
detect polyps.
The diffuse nature of the disease necessitates complete colectomy with ileo-anal
pull through.
FPC is autosomal dominant and if left untreated all patients develop colon
cancer by age 35-40.

Study Topic: Familial Adenomatous Polyposis


We update our content daily based on user feedback. Please click here to submit
your suggestions or if interested in becoming an author or editor, please contact us
at editor@medicalpearlspub.com.

Question 99: What is the typical adult dose of Tc99m-pertechnetate for


scrotal scintigraphy?

Choices:
1. 5-10 mCi
2. 5-20 mCi
3. 1-2 mCi
4. 20 mCi

Answer: [4] - 20 mCi


Explanations:
This varies from lab to lab, and normal recommendations have varied from 5 to
30 mCi.
20 mCi is a very common dose, and the correct answer here.

Study Topic: Testicle, Scintigram


We update our content daily based on user feedback. Please click here to submit
your suggestions or if interested in becoming an author or editor, please contact us
at editor@medicalpearlspub.com.

Question 100: A 75 year old with elevated cholesterol and atherosclerosis


develops right hemiparesis which over 6 hours progresses to hemiplegia. He is
awake and alert. What is the most likely diagnosis?

Choices:
1. Obstructive hydrocephalus
2. Large brain tumor with edema
3. CVA
4. Migraine

Answer: [3] - CVA


Explanations:
This most likely represents a thrombotic stroke. These sometimes have a gradual
onset.
A thrombotic stroke differs from an embolic stroke where the blockage is due to
material coming from a distant source, such as the heart.
Lacunar stroke is a thrombus developing in small arteries that mostly supply the
brainstem.
Many times it is difficult to differentiate between thrombotic and embolic CVAs.

Study Topic: Stroke, Acute (Cerebral Vascular Accident)


We update our content daily based on user feedback. Please click here to submit
your suggestions or if interested in becoming an author or editor, please contact us
at editor@medicalpearlspub.com.

Question 101: Which vascular finding contraindicates endovascular


embolization of a spinal AVM?

Choices:
1. The anterior spinal artery originates at the same level as the AVM
2. Patient is asymptomatic at AVM discovery
3. Multiple spinal arteries supply AVM nidus
4. Only one dominant venous outflow is noted to be present

Answer: [1] - The anterior spinal artery originates at the same level as the AVM
Explanations:
If attempts are made to endovascularly embolize an AVM supplied by the same
artery (anterior spinal) that supplies the cord, the cord could infarct.
Multiple spinal arteries supplying the AVM simply means multiple vessels must
be embolized
Given the risk of cord ischemia from an AVM, even asymptomatic patients
should be treated
Venous outflow does not negatively impact on success rate of endovascular
embolization therapy

Study Topic: Spinal Arteriovenous Malformation


We update our content daily based on user feedback. Please click here to submit
your suggestions or if interested in becoming an author or editor, please contact us
at editor@medicalpearlspub.com.

Question 102: In a patient with a diagnosis of Monckeberg sclerosis, which


of the following is most likely seen?

Choices:
1. Atherosclerotic plaques in the aorta
2. Narrowing of the coronary vessels
3. Calcified radial artery
4. Aneurysm of the abdominal aorta

Answer: [3] - Calcified radial artery


Explanations:
Monckeberg arteriosclerosis is also known as medial calcific sclerosis.
In this disorder calcium deposits in the middle layer (the tunica media) of the
wall of medium-sized vessels.
The disorder is benign and only leads to hardening of vessels. The lumen
remains patent.
The prevalence of Monckeberg arteriosclerosis is known to increase with age.
Known associations include diabetes, chronic renal disease, systemic lupus
erythematosus, chronic inflammatory disorders, and hypervitaminosis D.

Study Topic: Sclerosis, Monckeberg


We update our content daily based on user feedback. Please click here to submit
your suggestions or if interested in becoming an author or editor, please contact us
at editor@medicalpearlspub.com.

Question 103: Labeled RBCs are involved, or can be involved, in any of the
following studies EXCEPT

Choices:
1. Liver blood pool imaging
2. GI bleeding exam
3. Splenic imaging
4. Imaging for Meckel diverticulum

Answer: [4] - Imaging for Meckel diverticulum


Explanations:
Certainly if you're evaluating a blood pool distribution, imaging the RBCs in the
blood can help.
Although you can do GI bleeding exams with sulfur colloid, labeled RBCs are
better for intermittent bleeds.
Splenic imaging can be done with sulfur colloid as well, but denatured RBCs are
more specific.
NM does not image the Meckel diverticulum directly, as barium studies do, but
the majority of Meckel diverticula contain ectopic gastric mucosa, which takes
up Tc99m pertechnetate.

Study Topic: Red Blood Cell, Labeled Scans


We update our content daily based on user feedback. Please click here to submit
your suggestions or if interested in becoming an author or editor, please contact us
at editor@medicalpearlspub.com.

Question 104: Which of the following is correct about time of flight (TOF)
vs. Phase contrast (PC) MRA?

Choices:
1. Only PC can provide directional information
2. Only TOF can quantify velocity
3. Both 1 and 2 are true
4. Neither 1 nor 2 are true

Answer: [4] - Neither 1 nor 2 are true


Explanations:
Phase contrast can provide quantitative information about velocity and flow
direction, so Choice 2 is false.
While TOF without modification shows all flowing photons, independent of
direction, the application of a saturation band allows TOF to show flow in only
one direction, so Choice 1 is false.
Use of sat bands is how you can obtain MR arteriograms and MR venograms
instead of having both arterial AND venous flow on all images.

Study Topic: MR Angiogram


We update our content daily based on user feedback. Please click here to submit
your suggestions or if interested in becoming an author or editor, please contact us
at editor@medicalpearlspub.com.

Question 105: An 18 year old female without significant past medical


history presents with cervical, axillary, supraclavicular, and inguinal
lymphadenopathy. She has had some generalized pruritus, undocumented
fevers, and night sweats. She started coughing about 10 days ago but produces
no sputum. The lungs are clear and there is no hepatosplenomegaly. Select the
next step in management.

Choices:
1. Broad spectrum antibiotics
2. Chest x-ray
3. Serology for Bartonella bacilliformis
4. Lymph node biopsy

Answer: [2] - Chest x-ray


Explanations:
The patient most likely has Hodgkin lymphoma. The most important issue is to
get a lymph node biopsy. With the respiratory symptoms the patient should have
a chest x-ray, labs, and a PPD prior to the procedure.
The chest x-ray may show a mediastinal mass or pneumonia. Bartonella
bacilliformis serology would be appropriate if the diagnosis was Bartonellosis
(Cat scratch disease).
After the biopsy proves the diagnosis, PET/CT is done for staging purposes.
There is some debate as to whether a chest x-ray is needed, at all.
Hodgkin disease in developed countries is most common in late adolescence and
early adulthood

Study Topic: Cancer, Lymphoma, Hodgkin


We update our content daily based on user feedback. Please click here to submit
your suggestions or if interested in becoming an author or editor, please contact us
at editor@medicalpearlspub.com.

Question 106: What sign is not part of Charcot triad?


Choices:
1. Fever
2. Jaundice
3. Murphy sign
4. Hypotension

Answer: [4] - Hypotension


Explanations:
Charcot triad is indicative of cholangitis
Hypotension and mental status changes are part of Reynold pentad for
cholangitis

Study Topic: Cholangitis


We update our content daily based on user feedback. Please click here to submit
your suggestions or if interested in becoming an author or editor, please contact us
at editor@medicalpearlspub.com.

Question 107: The great cerebral vein of Galen and the inferior sagittal
sinus unite to form which of the following sinuses?

Choices:
1. Inferior petrosal sinus
2. Occipital sinus
3. Straight sinus
4. Cavernous sinus

Answer: [3] - Straight sinus


Explanations:
The straight or tentorial sinus is formed by the union of the great cerebral vein
and the inferior sagittal sinus.
It permits veins to run from the center of the head posteriorly to the confluence
of sinuses.
It is triangular in cross section and increases in size moving posteriorly.
The straight sinus, occipital sinus, and superior sagittal sinus combine to
terminate at the torcula Herophili, or confluence of sinuses. This confluence is
drained by right and left transverse sinuses.

Study Topic: Neuroanatomy, Brain, Sinuses, Dural Venous Sinuses


We update our content daily based on user feedback. Please click here to submit
your suggestions or if interested in becoming an author or editor, please contact us
at editor@medicalpearlspub.com.

Question 108: Which is not included in the middle ear?


Choices:
1. Stapes
2. Incus
3. Malleus
4. Semicircular canal

Answer: [4] - Semicircular canal


Explanations:
The three bones, malleus, incus, and staples are found in the middle ear.
The semicircular canal is part of the inner ear.
Oscillation of these three bones creates movement of the fluid inside the
labyrinth.
Movement of the fluid inside the labyrinth stimulates nerve endings of the
cochlear nerve.

Study Topic: Anatomy, Head, Ear, Middle Ear


We update our content daily based on user feedback. Please click here to submit
your suggestions or if interested in becoming an author or editor, please contact us
at editor@medicalpearlspub.com.

Question 109: Which of the following is the best measure of bone density?
Choices:
1. CT scan
2. DEXA scan
3. MRI scan
4. Serum calcium

Answer: [2] - DEXA scan


Explanations:
The dual-energy X-ray absorptiometry or DEXA is the best method to measure
bone density
DEXA can be done centrally (spine and hip) or peripherally (extremity)
Central testing is preferable

Study Topic: Dual Energy Xray Absorptiometry (DEXA)


We update our content daily based on user feedback. Please click here to submit
your suggestions or if interested in becoming an author or editor, please contact us
at editor@medicalpearlspub.com.

Question 110: Which one of the following is a precursor to melatonin?


Choices:
1. Glutamine
2. Serine
3. Tryptophan
4. Histidine

Answer: [3] - Tryptophan


Explanations:
Tryptophan is converted to serotonin that in turn is converted to melatonin
Melatonin is secreted from the pineal gland
Melatonin plays a roll in sleep and thermo regulation

Study Topic: Physiology, Melatonin


We update our content daily based on user feedback. Please click here to submit
your suggestions or if interested in becoming an author or editor, please contact us
at editor@medicalpearlspub.com.

Question 111: All of the following are true about lower gastrointestinal
hemorrhage with hemodynamic consequences except which of the following?

Choices:
1. Intussusception is a major cause in children age 3 months to 36 months
2. The primary cause of hemorrhage is carcinoma
3. The primary cause of significant hematochezia in patients over 60 years is
diverticulosis
4. Meckel diverticulum occurs at any age but happens more in children

Answer: [2] - The primary cause of hemorrhage is carcinoma


Explanations:
Colon cancer often causes bleeding but is rarely hemodynamically significant
While diverticular disease is the primary cause of patients over 60,
angiodysplasia is second
Significant hematochezia in children is most often caused by Meckel
diverticulum with intussusception being the second most common

Study Topic: Gastrointestinal Bleeding, Lower


We update our content daily based on user feedback. Please click here to submit
your suggestions or if interested in becoming an author or editor, please contact us
at editor@medicalpearlspub.com.

Question 112: Who is least at risk of iron deficiency anemia?


Choices:
1. Women with heavy menses
2. Elderly patients with chronic blood in their stools
3. Pregnant women
4. Males 15 to 25

Answer: [4] - Males 15 to 25


Explanations:
Iron deficiency anemia is most likely in menstruating females, the very old, and
the very young.
It can be due to inadequate intake or ongoing losses

Study Topic: Anemia, Iron Deficiency


We update our content daily based on user feedback. Please click here to submit
your suggestions or if interested in becoming an author or editor, please contact us
at editor@medicalpearlspub.com.

Question 113: A patient is diagnosed with a right middle cerebral artery


thrombosis. Which of the following would not be affected?

Choices:
1. Sight
2. Stereognosis
3. Motor
4. Pain and temperature

Answer: [1] - Sight


Explanations:
The posterior cerebral artery supplies the visual receptive area
The middle cerebral artery supplies the lateral brain
The precentral gyrus is involved with motor function, the postcentral gyrus is
involved with sensation, and the transverse temporal gyrus processes auditory
information

Study Topic: Stroke, Middle Cerebral Artery


We update our content daily based on user feedback. Please click here to submit
your suggestions or if interested in becoming an author or editor, please contact us
at editor@medicalpearlspub.com.

Question 114: A patient is hospitalized after a motor vehicle accident with


fracture of the pelvis. At 24 hours the patient develops shortness of breath,
fever, and crackles in the lung. Chest radiograph shows bilateral infiltrates.
Venous duplex of the lower extremities is negative. What is the most likely
diagnosis?

Choices:
1. Atelectasis
2. Pneumonia
3. Fat emboli
4. Pulmonary thromboembolism

Answer: [3] - Fat emboli


Explanations:
Fat embolism is a complication of fracture of the pelvis most often occurring at
24 to 48 hours post injury.
There is a 10 to 20 percent mortality rate.
Imaging of fat emboli is less sensitive than imaging of thromboemboli.
The embolic fat particles are smaller than embolic blood clots. Therefore, the
level of pulmonary vessel occlusion is more distal.

Study Topic: Embolism, Fat


We update our content daily based on user feedback. Please click here to submit
your suggestions or if interested in becoming an author or editor, please contact us
at editor@medicalpearlspub.com.

Question 115: Which of the following is rarely affected in Paget disease?


Choices:
1. Sacrum
2. Tibia
3. Humerus
4. Metatarsals

Answer: [4] - Metatarsals


Explanations:
Paget disease can affect almost every bone in the skeleton but has affinity for
long bones.
The hands and feet are very rarely involved.
The most common bones involved are pelvis, spine, femur, and skull.
Pagetoid bone goes through lytic, sclerotic, and mixed phases, each with
distinctive radiographic appearance. A long-term complication is rare malignant
degeneration.

Study Topic: Paget Disease


We update our content daily based on user feedback. Please click here to submit
your suggestions or if interested in becoming an author or editor, please contact us
at editor@medicalpearlspub.com.

Question 116: What preparation is necessary prior to a KUB (kidney,


ureter, bladder radiograph)?

Choices:
1. NPO (nothing by mouth) after midnight
2. Emptying the bladder just before the image
3. No preparation
4. Mild sedation beforehand

Answer: [3] - No preparation


Explanations:
The image is taken on a supine patient and requires no preparation.
KUBs are distinguished from abdominal radiographs. The former must include
the symphysis while the latter must include the diaphragms. In taller persons,
therefore, the KUB is centered lower than the abdominal radiograph.
KUBs were once thought sensitive for detection of urinary tract calculi,
reportedly detecting 85% of all such stones. Subsequent routine use of CT has
demonstrated that the KUB is poorly sensitive even to calcified stones evident on
advanced imaging.
Preparation is required for BE and IVP studies, of which the first step is to
obtain a KUB, but this is not the question asked. BEs require bowel prep and the
bladder should be emptied just prior to performing an IVP.

Study Topic: Radiographic Evaluation Of The Urinary Tract


We update our content daily based on user feedback. Please click here to submit
your suggestions or if interested in becoming an author or editor, please contact us
at editor@medicalpearlspub.com.

Question 117: What is the best method to image a loculated pleural


effusion?

Choices:
1. Decubitus chest radiographs (B/L)
2. Chest CT
3. Thoracoscopic visualization
4. Chest MRI

Answer: [2] - Chest CT


Explanations:
Chest CT is preferred for confirmation of a loculated pleural effusion.
Plain radiographs are inferior to CT or ultrasonography in pleural effusion
evaluation.
Decubitus chest radiographs are recommended for assessing small non-loculated
pleural effusions, but shouldn't help with regard to loculated pleural effusions.
CT or US can also be used to guide interventional procedures.

Study Topic: Effusion, Pleural


We update our content daily based on user feedback. Please click here to submit
your suggestions or if interested in becoming an author or editor, please contact us
at editor@medicalpearlspub.com.

Question 118: In which of the following will one encounter acute chest
syndrome?

Choices:
1. Atherosclerosis
2. Smoking addiction
3. Sickle cell anemia
4. Pulmonary hypertension

Answer: [3] - Sickle cell anemia


Explanations:
Acute chest syndrome is a non-infectious vasoocclusive crisis of the lung blood
vessels.
The disorder is often seen in sickle cell disease and can be serious.
The chest radiograph usually reveals a focus of air-space opacification. The crisis
is often precipitated by an infection and leads to sickling of red cells in the lung.
Death is common if treatment is delayed.

Study Topic: Chest Syndrome, Acute


We update our content daily based on user feedback. Please click here to submit
your suggestions or if interested in becoming an author or editor, please contact us
at editor@medicalpearlspub.com.

Question 119: On non-enhanced CT, you see an adrenal mass in a


hypertensive patient. Should you R/O pheochromocytoma with lab studies
prior to performing a contrast enhanced scan so as to avoid the risk of
inducing a hypertensive crisis?

Choices:
1. There have been several reported cases of this happening in recent years
2. Alpha blockade would be prudent in known pheochromocytoma patients, but
laboratory testing to "R/O pheo" on all hypertensives prior to giving IV contrast is
not reasonable
3. You should arrange with your laboratory to run STAT urine metanephrine levels to
deal with this uncommon but serious problem
4. Concern for pheochromocytoma should be assessed at the time of scheduling,
with pre-test labs run on all for whom the ordering physician indicates possible
concern

Answer: [2] - Alpha blockade would be prudent in known pheochromocytoma


patients, but laboratory testing to "R/O pheo" on all hypertensives prior to giving IV
contrast is not reasonable

Explanations:
Hypertensive crisis induced by IV iodinated contrast administration in a patient
with unblocked pheochromocytoma is extremely rare; many years have gone by
without a reported case.
In a KNOWN patient with pheochromocytoma, it is prudent to administer an
alpha blocker prior to giving iodinated contrast, but the vast majority of
hypertensive patients do NOT have pheochromocytoma, so blocking all
hypertensives is unrealistic.

Study Topic: Cancer, Chromaffin Cell (Pheochromcytoma)


We update our content daily based on user feedback. Please click here to submit
your suggestions or if interested in becoming an author or editor, please contact us

at editor@medicalpearlspub.com.

Question 120: A newborn does not pass meconium and starts vomiting all
feedings. She develops abdominal distention but the anus is patent.
Radiograph of the abdomen reveals no free air or luminal distension. Bubbly
feces are seen. Select the most likely diagnosis.

Choices:
1. Annular pancreas
2. Hirschsprung disease
3. Cystic fibrosis
4. Duodenal atresia

Answer: [3] - Cystic fibrosis


Explanations:
Meconium ileus can be the presentation of cystic fibrosis.
Initial treatment is with mucolytics and enemas.
Occasionally there can be perforation and bladder fistulas requiring surgery.
Annular pancreas and duodenal atresia would be associated with vomiting but
not failure to pass meconium. Hirschsprung is not typically associated with
neonatal vomiting of all feedings.

Study Topic: Cystic Fibrosis


We update our content daily based on user feedback. Please click here to submit
your suggestions or if interested in becoming an author or editor, please contact us
at editor@medicalpearlspub.com.

Question 121: During a subfalcine herniation, what structure is commonly


affected?

Choices:
1. 3rd cranial nerve
2. Anterior cerebral artery branches
3. Midbrain
4. Caudate nucleus

Answer: [2] - Anterior cerebral artery branches


Explanations:
In a subfalcine herniation, the cingulated gyrus on the medial aspect of the
frontal lobe displaces across under the falx across the midline.
This type of herniation affects the anterior cerebral arteries and its branches.
Subfalcine herniation does not affect the brain stem to a large degree.
CN III is injured during transtentorial, not subfalcine, herniation

Study Topic: Herniation, Subfalcine


We update our content daily based on user feedback. Please click here to submit
your suggestions or if interested in becoming an author or editor, please contact us
at editor@medicalpearlspub.com.

Question 122: 18-FDG PET imaging is very helpful in all BUT...


Choices:
1. Colorectal cancer re-staging
2. Treatment monitoring of non-Hodgkin lymphoma
3. Assessing for breast cancer metastases
4. Detecting H. pylori

Answer: [4] - Detecting H. pylori


Explanations:
H. pylori is detected using a radioactive carbon breath test.
PET has a role in the other three scenarios.

Study Topic: PET Scanning


We update our content daily based on user feedback. Please click here to submit
your suggestions or if interested in becoming an author or editor, please contact us
at editor@medicalpearlspub.com.

Question 123: What is the most common cause of pathologic bowing of the
legs?

Choices:
1. Blount disease
2. Achondroplasia
3. Rickets
4. Slipped capital femoral epiphyses

Answer: [1] - Blount disease


Explanations:
Blount disease is the most common cause of pathologic bowing of the legs
It is most commonly seen in ages 1 to 3
It commonly occurs in overweight children who walk at an early age
A growth disturbance involving the proximal tibial epiphysis causes sharp
angulation at the proximal tibia

Study Topic: Blount Disease


We update our content daily based on user feedback. Please click here to submit
your suggestions or if interested in becoming an author or editor, please contact us
at editor@medicalpearlspub.com.

Question 124: Facial palsy is present in a patient with a probable temporal


bone fracture. What type of fracture is most likely?

Choices:
1. Transverse
2. Longitudinal
3. Oblique
4. Mixed

Answer: [2] - Longitudinal


Explanations:
Although transverse temporal bone fractures more likely cause a facial palsy,
longitudinal fractures of the temporal bone are far more common and can also
cause facial palsy.
So given facial palsy, longitudinal temporal bone fractures are more likely.

Study Topic: Trauma, Bone, Facial Nerve Intratemporal


We update our content daily based on user feedback. Please click here to submit
your suggestions or if interested in becoming an author or editor, please contact us
at editor@medicalpearlspub.com.

Question 125: The majority of breast carcinomas are found:


Choices:
1. In the upper outer quadrant of the breast
2. In the lower half of the breast
3. Around the areola
4. In the medial half

Answer: [1] - In the upper outer quadrant of the breast


Explanations:
More than half of all breast carcinomas are found in the upper outer quadrant of
the breast.

Study Topic: Cancer, Breast


We update our content daily based on user feedback. Please click here to submit
your suggestions or if interested in becoming an author or editor, please contact us
at editor@medicalpearlspub.com.

Question 126: What is the most common cause of malignant pleural


effusion?

Choices:
1. Lung cancer in both males and females
2. Lung cancer in males and breast cancer in females
3. Lung cancer in males and ovarian cancer in females
4. Gastrointestinal cancers in males and breast cancer in females

Answer: [2] - Lung cancer in males and breast cancer in females


Explanations:
Breast cancer metastasis is the most common cause of malignant pleural
effusions in females.
Ovarian cancer can cause pleural effusions but it is not the most common cause.
Gastrointestinal cancers are a less common cause.
Malignant pleural effusions often appear bloody.

Study Topic: Malignant Effusion


We update our content daily based on user feedback. Please click here to submit
your suggestions or if interested in becoming an author or editor, please contact us
at editor@medicalpearlspub.com.

Question 127: Patients that are intravenous drug users are at a higher risk
to develop:

Choices:
1. Coronary artery disease
2. Epidural abscess
3. Clostridium difficile
4. Varicella

Answer: [2] - Epidural abscess


Explanations:
An epidural abscess can develop in the epidural space from systemic spread of
blood borne infection

Study Topic: Abscess, Epidural


We update our content daily based on user feedback. Please click here to submit
your suggestions or if interested in becoming an author or editor, please contact us
at editor@medicalpearlspub.com.

Question 128: You elute 630 mCi Tc99m from a Mo/Tc generator. What is
the NRC limit of total Mo activity permitted?

Choices:
1. 0.15 microcurie
2. 94.5 microcurie
3. 47 microcurie
4. 94.5 microcurie/ml

Answer: [2] - 94.5 microcurie


Explanations:
A radiopharmaceutical may not contain more than 0.15 microcurie of 99Mo per
mCi of 99m Tc, the answer here is 0.15 x 630, or 94.5 microcurie.

Study Topic: Nuclear Medicine


We update our content daily based on user feedback. Please click here to submit
your suggestions or if interested in becoming an author or editor, please contact us
at editor@medicalpearlspub.com.

Question 129: Which bone fracture is presented in a nightstick fracture?


Choices:
1. Distal radius
2. Distal ulna
3. Proximal ulna
4. Proximal radius

Answer: [2] - Distal ulna


Explanations:
Galeazzi fracture is a fracture of the distal radius
Nightstick fracture is a fracture of the distal ulna secondary to a direct blow as
from a nightstick
Monteggia fracture is a fracture of the proximal ulna
Hutchinson fracture is a fracture of the distal radius

Study Topic: Fracture, Nightstick


We update our content daily based on user feedback. Please click here to submit
your suggestions or if interested in becoming an author or editor, please contact us
at editor@medicalpearlspub.com.

Question 130: With regard to parathyroid imaging with Tc99m sestamibi,


which of the following statements is TRUE of 2-3 hour delayed images?

Choices:
1. Thyroid activity persists, but parathyroid activity shows complete washout
2. Parathyroid adenoma activity persists, but thyroid activity fades
3. Hyperfunctioning thyroid activity persists; no activity in parathyroid
4. Normal parathyroid glands are routinely visualized

Answer: [2] - Parathyroid adenoma activity persists, but thyroid activity fades
Explanations:
Normally, parathyroid adenomas and normal thyroid are both seen on early
images, but on delayed images, the thyroid washes out and the parathyroid
adenomas remain active.
Normal parathyroid activity is too small to be routinely seen, even on delayed
images.
Hyper-functioning thyroid foci can remain active on delayed images, a potential
false positive.

Study Topic: Cancer, Parathyroid, Adenoma


We update our content daily based on user feedback. Please click here to submit
your suggestions or if interested in becoming an author or editor, please contact us
at editor@medicalpearlspub.com.

Question 131: How often does imaging the liver in patients with cancer
elsewhere show a focal liver lesion < 1 cm in diameter?

Choices:
1. < 1% of the time
2. 6% of the time
3. 13% of the time
4. 38% of the time

Answer: [3] - 13% of the time


Explanations:
The actual answer is 13%.
The differential is liver cyst, biliary hamartoma, hemangioma, and metastatic
deposit.
Even with a known primary malignancy, if the lesions are small, multiple, and
UNIFORM in size, biliary hamartomas (von Meyenburg complexes) are favored.
The fact that 13% is not rare is indicative of why managing/imaging
"incidentalomas" has become a major issue in radiology.

Study Topic: Incidentaloma


We update our content daily based on user feedback. Please click here to submit
your suggestions or if interested in becoming an author or editor, please contact us
at editor@medicalpearlspub.com.

Question 132: Onion skinning on a bone radiograph is indicative of which


condition?

Choices:
1. Osteogenic sarcoma
2. Chondroma
3. Ewing sarcoma
4. Osteoma

Answer: [3] - Ewing sarcoma


Explanations:
Ewing sarcoma is common in children and often occurs in long bones and the
pelvis.
Typical features of presentation include fever, pain, weight loss, anemia, and
leukocytosis.
Radiographs may show onion skinning due to successive layers of periosteal
development. This may be present with osteogenic sarcoma as well, but it is
more likely seen in Ewing sarcoma.
Other features on radiography may include moth eaten appearance and soft tissue
mass with saucerization.

Study Topic: Cancer, Sarcoma, Ewing


We update our content daily based on user feedback. Please click here to submit
your suggestions or if interested in becoming an author or editor, please contact us
at editor@medicalpearlspub.com.

Question 133: Schizencephaly is also known as "cleft brain." What lines the
cleft in such patients?

Choices:
1. CSF
2. Gray matter
3. Hamartomatous tissue
4. White matter

Answer: [2] - Gray matter


Explanations:
Schizencephaly clefts are lined by gray matter, of either the heterotopic, pachyor polymicrogyric variety
The clefs extend from the pial surface laterally to the ependymal lateral
ventricular lining medially
Large primitive veins may overlie the cleft
Porencephaly also has clefts, but these are due to trauma or vascular insult, and
thus lined by gliotic white matter, not dysplastic gray matter

Study Topic: Schizencephaly


We update our content daily based on user feedback. Please click here to submit
your suggestions or if interested in becoming an author or editor, please contact us
at editor@medicalpearlspub.com.

Question 134: You view a brain MRI sequence with a short TR and short
TE. Which is correct when comparing gray and white matter?

Choices:
1. White matter is hyperintense to gray matter
2. White matter is hypointense to gray matter
3. White matter is hypointense to CSF
4. White matter is isointense to gray matter

Answer: [1] - White matter is hyperintense to gray matter


Explanations:
The simple mnemonic "White is white on (wh)one" indicates white matter is
hyperintense ("white") on T1 weighted images. This is relative to gray matter.
It "follows" that white matter is dark (darker than gray matter, hypointense) on
T2 weighted images.
The hyperintensity on T1 of white matter is related to the myelin content. Thus it
does not apply to immature, neonatal brains in which myelinization is not
complete.

Study Topic: Magnetic Resonance Imaging (MRI)


We update our content daily based on user feedback. Please click here to submit
your suggestions or if interested in becoming an author or editor, please contact us
at editor@medicalpearlspub.com.

Question 135: What is the most common cause of lung cancer?


Choices:
1. Long-term exposure to tobacco smoke
2. Asbestos
3. Radon gas exposure
4. Air pollution

Answer: [1] - Long-term exposure to tobacco smoke


Explanations:
The most common cause of lung cancer is long-term exposure to tobacco.

Study Topic: Cancer, Lung


We update our content daily based on user feedback. Please click here to submit
your suggestions or if interested in becoming an author or editor, please contact us
at editor@medicalpearlspub.com.

Question 136: Which of the following is the best screening test for autosomal
dominant polycystic kidney disease?

Choices:
1. Plain abdominal radiograph
2. Ultrasound
3. Intravenous pyelogram
4. Abdominal CT

Answer: [2] - Ultrasound


Explanations:
Ultrasound is the best screening test to check for autosomal dominant polycystic
kidney disease in family members of affected individuals.
The test is portable, non-invasive, and inexpensive.
Sensitivity is low in early disease and in young patients.
Abdominal CT is more accurate but requires ionizing radiation and is more
expensive. It is preferred in a more detailed workup of patients with symptoms
worrisome for complications of autosomal dominant polycystic kidney disease.
Renal MRI eliminates the concern for ionizing radiation and is also an excellent
diagnostic modality.

Study Topic: Polycystic Kidney Disease


We update our content daily based on user feedback. Please click here to submit
your suggestions or if interested in becoming an author or editor, please contact us
at editor@medicalpearlspub.com.

Question 137: Which is correct about second look US for lesions noted on
breast MRI?

Choices:
1. US finds a corresponding lesion in about half the cases
2. US finds corresponding masses more commonly than corresponding non-mass
lesions
3. US sensitivity for detecting corresponding masses increases with lesion size
4. All of the above are true

Answer: [4] - All of the above are true


Explanations:
Combining the data of 5 studies (N = 1024), US found the MRI-detected lesion
51% of the time.
A 2009 study showed corresponding US findings in MRI detected masses 62%
of the time, but only 31% of the time for non-mass MR findings.
US detects 50% of the MR masses < 5 mm in diameter, 73% of those 11-15 mm
in size, and 86% of MR detected masses > 15 mm.

Study Topic: Magnetic Resonance Mammography


We update our content daily based on user feedback. Please click here to submit
your suggestions or if interested in becoming an author or editor, please contact us
at editor@medicalpearlspub.com.

Question 138: In a supine position, aspiration will most likely affect which
lobe of the lung?

Choices:
1. Apical segment of the right lower lobe
2. Apical segment of the left lower lobe
3. Posterior segment of the right upper lobe
4. Posterior segment of the left upper lobe

Answer: [3] - Posterior segment of the right upper lobe


Explanations:
When a patient aspirates in a supine recumbent position, the aspiration is most
commonly found in the posterior segment of the right upper lobe and the
superior segment of the right lower lobe.
The right mainstem is wider than the left mainstem bronchus.
There is no apical segment of the right or left lower lobe. There is an apical
segment of the right upper lobe and an apico-posterior segment of the left upper
lobe.
If the patient were to aspirate while prone recumbent, the anterior segment of the
right upper lobe is commonly involved.

Study Topic: Pneumonia, Aspiration


We update our content daily based on user feedback. Please click here to submit
your suggestions or if interested in becoming an author or editor, please contact us
at editor@medicalpearlspub.com.

Question 139: Which of the following vascular structures are


intrapericardial in location?

Choices:
1. Main pulmonary bifurcation
2. Junction of middle and inferior pulmonary veins on the right
3. Right interlobar artery
4. Azygous arch

Answer: [1] - Main pulmonary bifurcation


Explanations:
The main pulmonary arterial bifurcation into right and left main branches is
within the pericardium
The other listed structures are extrapericardial in location

Study Topic: Anatomy, Thorax, Heart


We update our content daily based on user feedback. Please click here to submit
your suggestions or if interested in becoming an author or editor, please contact us
at editor@medicalpearlspub.com.

Question 140: When does data collection stop during a MUGA scan?
Choices:
1. When a predetermined number of counts is acquired
2. When a predetermined number of cardiac cycles has been acquired
3. Options 1 and 2
4. When a preset time has been reached

Answer: [3] - Options 1 and 2


Explanations:
Gated partitioning of the cardiac cycle ends when a preset # of counts or cardiac
cycles has been reached.
Counts and cardiac cycles discarded due to arrhythmias are not counted.

Study Topic: Radionuclide Ventriculography


We update our content daily based on user feedback. Please click here to submit
your suggestions or if interested in becoming an author or editor, please contact us
at editor@medicalpearlspub.com.

Question 141: Regarding the appearance of an infarct on MRI, which of the


following is true?

Choices:
1. A hyperacute infarct will appear hyperintense on DWI, T2WI and hypointense on
ADC
2. A chronic infarct will appear hypointense on T2WI and hyperintense on DWI and
ADC
3. A subacute infarct will appear hypointense on T1WI and hyperintense on T2WI
and ADC
4. An acute infarct will appear hyperintense on T1WI, T2WI and DWI

Answer: [1] - A hyperacute infarct will appear hyperintense on DWI, T2WI and
hypointense on ADC

Explanations:
Areas of infarct express cytotoxic edema (shift of water into cells), which leads
to restricted diffusion of water molecules primarily due to dysfunction of
membrane pumps and damage to intracellular structures.
During the hyperacute and acute stages of ischemic stroke DWI should
demonstrate a focal area of restricted diffusion (area of high signal) and the ADC
should demonstrate a focal area of low intensity.
In the subacute and chronic stages there is hyperintense signal on DWI primarily
due to T2 shine through.
Moving through the subacute phase, the high DWI signal is over time
increasingly due to T2 "shine through," as the ADC moves from hypointense to
hyperintense.

Study Topic: Stroke, Imaging


We update our content daily based on user feedback. Please click here to submit
your suggestions or if interested in becoming an author or editor, please contact us
at editor@medicalpearlspub.com.

Question 142: Which of the following is not commonly associated with


cavernous malformations?

Choices:
1. Developmental venous anomaly
2. Dural arteriovenous fistula
3. Hyperkeratotic capillary-venous malformations
4. Superficial siderosis

Answer: [2] - Dural arteriovenous fistula


Explanations:
Cavernous malformations are not commonly associated with DAVF though they
are commonly mistaken as such.
Benign vascular hamartomas, which can occur throughout the CNS.
T2 Gradient Echo sequence is the most optimal sequence for detection, they are
commonly missed on noncontrasted CT.
Commonly described on imaging as a "popcorn ball" due to its heterogenous
appearance with hemorrhage of different ages.

Study Topic: Cerebral Cavernous Malformations


We update our content daily based on user feedback. Please click here to submit
your suggestions or if interested in becoming an author or editor, please contact us
at editor@medicalpearlspub.com.

Question 143: A 34-year-old male undergoes an intravenous pyelogram for


vague left flank pain. and hematuria. The study reveals multiple focal
dilations of contrast in the renal medullae with striated cavities and tubular
dilatation. The radiologist describes these as having a "bunch of grapes"
appearance. The radiologist is thereby suggesting which of the following
diagnoses?

Choices:
1. Autosomal dominant polycystic kidney disease (ADPCKD)
2. Medullary sponge kidney (MSK)
3. Acute pyelonephritis
4. Renal angiomyolipomas (AML)

Answer: [2] - Medullary sponge kidney (MSK)


Explanations:
MSK is a developmental disorder in the medullary pyramids of the kidney and is
best diagnosed with excretory urography.
The disorder is characterized by numerous cystic dilatations of the collecting
tubules, described as a "bunch of grapes" or "bouquet of flowers."
MSK is usually asymptomatic, but may have complicated calculus disease. It is
found in about 1% of the population.
The disorder predisposes one to infection, stone formation, and hematuria. The
diagnosis can also be made using CT urography and MRI.

Study Topic: Medullary Sponge Kidney


We update our content daily based on user feedback. Please click here to submit
your suggestions or if interested in becoming an author or editor, please contact us
at editor@medicalpearlspub.com.

Question 144: Which of the following modalities has almost 100%


sensitivity and specificity for detecting complete rotator cuff tears?

Choices:
1. MRI
2. CT arthrography
3. Ultrasonography
4. Plain radiography

Answer: [2] - CT arthrography


Explanations:
The ability to evaluate the labrum, the long head of the biceps tendon, the
glenohumeral ligaments, and bony structures, as well as the presence of loose
bodies, makes CT arthrography a useful study
However, it is limited in the evaluation of partial tears, for which its sensitivity
drops dramatically

Study Topic: Rotator Cuff, Injury


We update our content daily based on user feedback. Please click here to submit
your suggestions or if interested in becoming an author or editor, please contact us
at editor@medicalpearlspub.com.

Question 145: Facts about acute subdural hematoma include all but which
of the following?

Choices:
1. Mortality rate is 50 to 70%
2. The blood is is between the dura and the arachnoid membranes
3. It is often associated with damage to the brain parenchyma
4. The condition is caused by arterial bleeding

Answer: [4] - The condition is caused by arterial bleeding


Explanations:
Subdural hematoma is usually the result of venous bleeding from bridging veins
between the arachnoid membranes and the dura
Subarachnoid bleeding is more common than either subdural bleeding
Mortality rates have been estimated from 30 to 90% but is probably around 60%

Study Topic: Subdural Hematoma


We update our content daily based on user feedback. Please click here to submit
your suggestions or if interested in becoming an author or editor, please contact us
at editor@medicalpearlspub.com.

Question 146: You see a cystic breast lesion with uniformly thickened wall
and no hyperemia. This is most likely...

Choices:
1. An inflamed cyst
2. A neoplastic cyst
3. A cyst with fibrotic wall
4. Cannot tell based on information provided

Answer: [3] - A cyst with fibrotic wall


Explanations:
Since there is no hyperemia, it is likely NOT an inflamed cyst.
Since the wall thickening is uniform, it is likely NOT a neoplastic cyst.
Fibrosis is the end result of inflammation. Such cysts were previously inflamed
cysts that subsequently resolved.

Study Topic: Breast, Fibrocystic Disease


We update our content daily based on user feedback. Please click here to submit
your suggestions or if interested in becoming an author or editor, please contact us
at editor@medicalpearlspub.com.

Question 147: Which of the following radiographs is most often used in


assessing bone age?

Choices:
1. Lateral skull
2. PA CXR
3. AP Knee
4. PA hand

Answer: [4] - PA hand


Explanations:
A PA view of the left hand and wrist is most often used to evaluate skeletal
maturation, or "bone age."
Interpretation is based on comparison with standard views based on age and
gender.
Other sites can be used. For example, an atlas of bone age based on knee
radiographs is available.

Study Topic: Bone, Age


We update our content daily based on user feedback. Please click here to submit
your suggestions or if interested in becoming an author or editor, please contact us
at editor@medicalpearlspub.com.

Question 148: How can bone scintigraphy distinguish rib fractures from
thoracic bone metastases?

Choices:
1. Rib fractures are cold spots; bone mets are hot spots
2. Rib fractures are grouped; bone mets are randomly spread
3. Rib fractures show as short hot foci; bone mets may also show as elongated linear
increased rib uptake
4. Options 2 and 3 but not 1

Answer: [4] - Options 2 and 3 but not 1


Explanations:
Both rib fractures and bone mets are "hot," take up more than the normal amount
of tracer. This will fade over months to years with bone fractures, depending in
part on whether or not the fracture is in a weight-bearing bone.
Statements 2 and 3 are both true.

Study Topic: Bone, Scan


We update our content daily based on user feedback. Please click here to submit
your suggestions or if interested in becoming an author or editor, please contact us
at editor@medicalpearlspub.com.

Question 149: What is the next step of management in a pregnant patient


with suspected pulmonary embolism?

Choices:
1. Order a perfusion scan
2. Spiral chest CT
3. Lower extremity duplex
4. Start oral warfarin

Answer: [1] - Order a perfusion scan


Explanations:
V/Q scan is the best modality to rule out PE in pregnancy
CT is to be avoid when possible in pregnancy
Warfarin is contraindicated because it crosses placenta

Study Topic: Embolism, Pulmonary


We update our content daily based on user feedback. Please click here to submit
your suggestions or if interested in becoming an author or editor, please contact us
at editor@medicalpearlspub.com.

Question 150: Ascariasis infection in the GI tract:


Choices:
1. Affects 10% of the world's population.
2. Is, in the United States, most commonly seen along the coast in the Southeast
3. Worm migration can involve the biliary tree and pancreatic duct
4. These parasites appear on barium studies as variable-diameter ribbon-like
intraluminal filling defect

Answer: [3] - Worm migration can involve the biliary tree and pancreatic duct
Explanations:
The most common parasitic infection in humans, it affects 25% of the world's
population
In the USA, the most common region of infection is the Southwest
Ascariasis can involve biliary tree, pancreatic duct, and GI tract; a large bolus
can obstruct the ileocecal valve
Variable diameter ribbon-like filling defects are flatworms, the apparent diameter
being a function of projection. Ascariasis is a roundworm, and appears therefore
as a uniform thickness filling defect.

Study Topic: Ascariasis


We update our content daily based on user feedback. Please click here to submit
your suggestions or if interested in becoming an author or editor, please contact us
at editor@medicalpearlspub.com.

You might also like